Sunteți pe pagina 1din 35

EUNACOM RESUMEN TRAUMATOLOGIA

TRAUMATOLOGIA1. FRACTURAS
Repaso EMN Traumatología Rotura del tendón de Aquiles: Clásica historia del dolor piedrazo en
Las fracturas dobles de mandíbula tienen riesgo de producir asfixia. el talón. Defecto palpable en la región de inserción del gastrocnemio.
La TVP sin profilaxis llega a un 88% de incidencia. Con heparinas no Thompson + (si apreto el gastrocnemio no se mueve el pié), flexión
fraccionadas baja a un 50% y con HBPM bajan a 25%. Las que plantar puede ser mantenida parcialmente.
embolizan son las proximales. 2/3 de los TEP mueren antes de los 30
minutos. Túnel carpiano: Atrapamiento más frecuente, involucra nervio
mediano. Síntomas vagos, predominio nocturno, urente, congestivo,
área del pulgar, índice y mitad del dedo medio.
Se ve también durante el embarazo, lactancia e hipotiroidismo. Dg
SD. COMPARTIMENTAL:
es con electromiografía. Examen físico signo de Phalen (S 75% E
-TIenen aumento de la presión, dolor con ROM pasivo, parestesias o
47%), signo de Tinel percusión en región túnel genera parestesias.
anestesias, paresia o plejia, pulsos ausentes (los últimos 3 son
Signo de Durkan es la compresión de la zona del tunel del carpo
tardíos). El dolor es progresivo, desproporcionado a los hallazgos. La
reproduciendo síntomas, con alta S y E.
medición de la presión se hace con un transductor de PA (se toman 3
mediciones porque hay fenómenos de seudocompartimentalización,
Fracturas + frc: radio distal (Colles) > cadera > vértebra lumbar
o sea hay presiones distintas en distintas regiones de la
extremidad).Lo más frecuente es en la pierna y antebrazo.
Fractura del temporal: Se clasifican en transversal, oblicua y CAPÍTULO TraumatologíaTraumato:
longitudinal.
-Las transversales tienen hemotimpano, sordera sensorioneural,
pérdida de la función vestibular. Un paciente de 15 años con dolor tibial (que empeora por la noche y
La fractura del escafoides se caracteriza por dolor en la mejoracon la aspirina) presenta en la radiografía una pequeña
tabaquera anatómica, dolor a la compresión axial del pulgar zona hipodensa rodeada por formación de hueso reactivo. ¿Cuál
(mayor S y E), dolor a la palpación del tubérculo del escafoides.. 1- es el diagnóstico más probable?
4% población general, 80-90% hombres, promedio 25 años, 60-90& Osteoma osteoide, que es un tumor benigno formador de hueso. Se
fracturas del carpo, 5-12% tienen otras fracturas, 1% de los casos observa clásicamenteen niños mayorcitos y adolescentes;
son bilaterales. Caída con apoyo de la mano, puede fracturase predomina en los hombres (relación varón-mujer, 2:1).
La mayor parte de los niños presentan dolor localizado, por regla
muñeca, cúpula radial, luxación humeral, disyunción AC.
general en el fémur y la tibia; sin embargo, también pueden
estar afectados los brazos y las vértebras. Las radiografías y la
Escoliosis: Signo inequívoco en el diagnóstico es la giba costal. La
tomografía computarizada (TC) muestran una zona osteolítica
escoliosis se define como una deformidad rotacional de 10º con
rodeada por un hueso reactivo densamente esclerótico; en la
rotación de los cuerpos vertebrales. Tienen deformidad en el plano
gammagrafía ósea se aprecian «manchas calientes».La lesión
coronal, hipoxifosis torácica, rotación de cuerpos vertebrales,
suele tener menos de 1 cm de diámetro, y se origina en la
patrones de curvas torácicas.
unión de las cortezas antiguay nueva. En el examen
anatomopatológico, la lesión es un tejido fibroso muy
Displasia de cadera: El mejor examen es la ecografía al mes, pero el
vascularizado,con una matriz osteoide y espículas óseas poco
más difundido y disponible es la radiografía de pelvis AP a los 3
calcificadas, que está rodeado por unadensa zona de hueso
meses. 0.3 a 3% población. 1 en cada 500 RNV. FR parto en nalgas,
esclerótico. El tratamiento consiste en la extirpación quirúrgica
sexo femenino, primiparidad, antecedentes familiares, otras
de lalesión.
malformaciones asociadas. El ángulo acetabular en RN es menor a
30º, 30-36 dudoso, >36 patológico. Disminuye 1º por mes. 20º al
año, 0º a los 4 años. Se usan las correas de Pavlik
¿Cuáles son los tipos de fracturas más frecuentes en los niños?
Radiculopatía: Las fracturas más frecuentes en los niños son las de fisis y metáfisis;
- TEPE: Test de extremidad extendida elevada, se debe estas fracturas son clásicamente pediátricas. Éstas son
producir dolor más allá de la rodilla (si no llega hasta la rodilla puede localizaciones en las que los huesos del niño son muy débiles y
ser cualquier otra causa excepto radiculopatía). no ha finalizado aún el proceso de osificación. También son
- Lasegue: Flexión de la cadera y luego extensión de la frecuentes las «fracturas en hebilla
rodilla. » (compresión) y las «fracturas en tallo verde».
- Gowers
¿Cuáles son los signos del síndrome del compartimental?
- Oconnell evalúa L2. En el síndrome compartimental inminente o ya establecido (en el
- L5 no pueden caminar de talones, sensibilidad interortejo. que la tumefacción causa isquemia distal) se observan las
«cinco P» del punto anterior. Entre estos signos, hay que
Principal factor pronóstico en fx expuestas: Mecanismo de la lesión, sospechar
TRAUMATOLOGIA
daño partes blandas, configuración de la fractura y el grado de el diagnóstico si existe un dolor intenso durante la tracción pasiva
contaminación. de los dedos (ya sea en flexión o en extensión). Una vez que
Tipo 3 automáticas: >6 horas de evolución, falta de cobertura partes el nervio se encuentra muy lesionado, el paciente ya no
blandas, lesión vascular que requiere reparación, arma de fuego, refiere dolor. Los pacientes inconscientes no reconocen a
segmentarias (alta energía), desastres naturales, amputaciones menudo el síndrome compartimental.
traumáticas. Debe tenerse un alto índice de sospecha en los pacientes con
lesiones graves y alteracióndel estado mental. Asimismo, en
Fractura osteoporótica más frecuente: Vertebral toracolumbar. ocasiones es muy difícil examinar a un lactante o niño pequeño
Osteoporosis se ve más en mujeres, las 3 más frecuentes son la asustados. Si existen dudas acerca de la presencia de este
extremidad distal del radio, vértebras y cadera. Tipo I (idiopática) síndrome, han de medirse las presiones del compartimento.
pérdida ósea trabecular, postMNP, 55-65 años, fx vertebrales. Tipo 2
pérdida cortical, sobre 70, más en mujeres, se fractura más la ¿Cuál es el tratamiento del síndrome compartimental?
cadera y extremo distal del radio. También están las secundarias con El síndrome compartimental es una urgencia. Las presiones del
sus mil y un causas. compartimento pueden disminuirse mediante incisión de la piel
y la fascia de los compartimentos afectados (p. ej.,
Caídas de pie: Pensar en fx de calcáneo y columna toracolumbar, anterior,lateral, posterior profundo, posterior superficial de la
también platillos tibiales (difícil que se fracture la cadera). Calcáneo pierna). La herida se deja abierta, y secubre con un apósito
TRAUMATOLOGIA

60% fracturas asociadas (10% columna, 7% bilaterales, 25% otras estéril hasta que disminuya la tumefacción. Los cambios de
fracturas EEII, 75% compromiso subastragalino). Principal apósito y el cierre parcial de la herida suelen hacerse en el
complicación a largo plazo es el dolor crónico. quirófano. En ocasiones, son necesarios injertos cutáneos.
¿Cómo se trata una fractura clavicular simple?

1
EUNACOM RESUMEN TRAUMATOLOGIA
El mejor tratamiento de estas fracturas consiste en colocar un 2. En los niños (al contrario que en los adultos), la diseminación de
cabestrillo y limitar la actividad.Aunque la fractura se une en 2- los adultos aparece más por vía hematógena que por
4 semanas, el cabestrillo puede retirarse cuando el niño se traumatismos locales.
sienta cómodo. La abolladura residual (callo de fractura) puede 3. En los niños con una herida por punción y osteomielitis, hay que
tardar hasta 2 semanas en alisarse (remodelarse). La cirugía pensar en el diagnóstico de infección por Pseudomonas
sólo es necesaria si la fractura es abierta, si hay lesión aeruginosa.
neurovascular o si existen trastornos vasculares cutáneos a 4. Debido a los infartos, los pacientes con drepanocitosis presentan
causa de un desplazamiento significativo. un alto riesgo de osteomielitis(especialmente, de infecciones
por Salmonella).
¿Qué tipo de fracturas presentan a menudo 5. En las radiografías, los signos óseos pueden no aparecer hasta
los niños que caen con los brazos extendidos? pasados 10-15 días.
Fracturas de Colles. 6. El recuento de leucocitos puede ser normal hasta en el 50% de los
pacientes con osteomielitis.

¿Cómo se mide la escoliosis según el método de Cobb?


Ésta es la técnica estándar utilizada para cuantificar la escoliosis en
Es un grupo de fracturas completas del radio distal, asociadas a un
las radiografías posteroanteriores
desplazamiento variable del fragmento distal. En la
o laterales. Se traza una línea a lo largo de la vértebra inclinada
exploración, la caída (con la mano extendida, la muñeca en
situada más hacia arriba de la curva, y otra en la situada más
flexión dorsal y el antebrazo en pronación) provoca a menudo
abajo. La curvatura está representada por el ángulo «a»,que
la observación de una clásica «deformidad en tenedor» de la
puede medirse de dos formas
muñeca.
PUNTOS CLAVE: ESCOLIOSIS
¿En qué consisten las maniobras de Ortolani y de Barlow? 1. Aunque una escoliosis superior a 10° es relativamente frecuente
Los métodos clínicos más fiables de detección siguen siendo la
(1-2%), es más rara la progresión a más de 25° con necesidad
maniobra de reducción de Ortolani y la maniobra de
de tratamiento.
provocación de Barlow. El lactante debe estar en decúbito 2. La ortosis no elimina la curvatura, pero puede prevenir su
supino y tranquilo. Ambas maniobras comienzan con lmas progresión.
caderas flexionadas a 90°. Para realizar la maniobra de 3. Es importante establecer el nivel de madurez óseo, puesto que el
Ortolani, se abduce la cadera y se eleva suavemente el riesgo de progresión aumenta con la inmadurez.
trocánter. De este modo,la cabeza del fémur luxada se desliza 4. Las curvas progresivas son siete veces más frecuentes en las
nuevamente hacia el interior del acetábulo chicas que en los chicos.
La maniobra de Barlow se realiza por aducción de la cadera 5. No todas las escoliosis son idiopáticas: evaluar la diferencia de
flexionada, empujando luego elmuslo hacia atrás en un intento longitud de las extremidades, los signos neurocutáneos y las
de provocar la luxación de la cabeza del fémur alteraciones neurológicas (especialmente, los reflejos).

¿Qué otros signos diagnósticos son sugestivos de DC? Placas y tornillos Estabilidad absoluta.
■ Asimetría del muslo y los pliegues glúteos: sin embargo, los Fracturas articulares.
pliegues pueden observarse Fracturas de antebrazo.
también en el 10% de los lactantes normales. Clavos Estabilidad dinámica.
■ Prueba de Galeazzi: con las caderas flexionadas a 90°, las rodillas Fracturas diafisarias de huesos largos.
pueden estar a niveles Agujas Estabilización de pequeños fragmentos. Fracturas
diferentes (a causa del aparente acortamiento femoral en un lado de infantiles (respetar la fisis).
la luxación asimétrica). Obenque Fracturas de olécranon y rótula.
■ Prueba de Allis: con las caderas flexionadas y los talones sobre la Transforma fuerzas de tracción en fuerzas de compresión (principio
mesa de exploraciones, del tirante).
una rodilla desnivelada es sugestiva de diagnóstico de luxación de Fijadores externos Fracturas infectadas.
cadera. Fracturas abiertas grados IIIB y IIIC. Estabilización fracturas de
■ Marcha de ánade, hiperlordosis de la columna lumbar: se observa pelvis.
Estabilización rápida de politraumas.
en pacientes mayores
con luxaciones bilaterales.

¿Cuál es la causa más frecuente de cadera dolorosa en un niño de


menos de 10 años? Artrosis:Clínica típica de artrosis, signos de artrosis (nódulos de
Sinovitis transitoria aguda; este trastorno también ha sido
bouchard y Herberden). Tratamiento inicial= Paracetamol. De
denominado sinovitis tóxica, cadera irritable y coxitis fugaz. Se
ahí vamos subiendo. Las fracturas intracapsulares se operan
trata de un trastorno inflamatorio autolimitado que aparece
porque producen ARTROSIS no PSEUDOARTROSIS.
antes de la adolescencia, no tiene causa conocida y,
generalmente, cursa con un pronóstico clínicobenigno. Sin
embargo, durante su evolución clínica, en ocasiones causa gran ➢ GES: tto en personas de 55 años o más, artrosis de cadera o TRAUMATOLOGIA
ansiedad entre rodilla, leve o moderada (prótesis solo en cadera?--> en artrosis
los médicos y los familiares del paciente, dado que puede simular de rodilla el ges cubre tratamiento médico, la protesis no es
otros trastornos de peor GES). Inicio tto dentro de 24 hrs de confirmado dg. Evaluación
pronóstico (como artritis séptica, lesiones de las partes blandas, por especialista dentro de 120 días desde derivación.
osteomielitis, enfermedad de
Legg-Calvé-Perthes [LCP], artritis reumatoide juvenil, deslizamiento
Displasia de Cadera
de la epífisis de la cabezafemoral o tumores). Aunque la
-Sospecha por FR: Oligoamnios, podálica, antecedente familiar 1°
sinovitis transitoria aguda puede manifestarse en cualquier
grado de displasia de cadera, etc.
momento (desde que el niño empieza a andar hasta el final de -o Signos: Ortolani, Barlow, Galeazzi (rodillas quedan asimétricas al
la juventud), la edad más frecuentede inicio es entre los 3 y los flectar las rodillas de la guagua con las plantas apoyadas).
6 años; asimismo, es más frecuente en los niños. La sinovitis -Screening: a los 3 meses con Rx de cadera - Si signos o factores de
transitoria aguda sigue siendo un diagnóstico de exclusión. El riesgo: Eco al mes y derivar.
tratamiento consiste en reposo y reducción de la sinovitis con -Tratamiento: Arnes de Pavlik.
antiinflamatorios. La mayoría de los pacientes muestra una -Principal causa de coxartrosis en pacientes jóvenes.
resolución completa de los síntomas antes de 2 semanas, y el
resto presenta a veces síntomas menos graves durante varias
Escoliosis
semanas. -Signo de Adams: Giba dorsal.
TRAUMATOLOGIA

->10 º para algún lado. Menos que eso no se habla de escoliosis.


PUNTOS CLAVE: OSTEOMIELITIS -menor a 25 grados no se hace nada.
1. En los niños sanos, los microorganismos causales más frecuentes Entre 25-40 corsé. Mas de 40 cirugía. Grosso modo.
son Staphylococcus aureus y estreptococo betahemolítico.

Infecciones osteoarticulares
2
EUNACOM RESUMEN TRAUMATOLOGIA
- Osteomielitis Aguda y AS: Agente etiológico mas frecuente: E. -Fractura de codo: nervio cubital: no separa dedos
Aureus. - Tratamiento típico: Cloxacilina.
Luxación del hombro
Lumbago tipico Signo de la charretera. Mecanismo por caída.
-Dg típico. Igual enfrentamiento si es dorsalgia o cervicalgia. Si no -Posterior es en epilépticos o electrocutados. Reducción de
tiene banderas rojas, siempre el tratamiento menos invasivo. emergencia.
Analgesia e imágenes. -Afecta nervio axilar (evaluar sensibilidad zona sobre deltoides, no
Lumbociatica radicular puede abducir)
-Examen de elección: RNM.
Pie plano
Luxofractura de tobillo
Hasta los 5 años es posible tener pie plano, recién ahì se comienza a
-Criterios de Otawa Politrauma
formar el arco plantar. -Siempre primero evaluar vía aérea.
-FR: Obesidad, usar zapatos. -Inmovilización cervical
-Si es flexible, manejo expectante. -Si paciente hipotenso o FC >120, transfundir hemoderivados.
-Si es rígido, derivar para manejo. -Set de Rx de Politrauma: Cervical Lateral, Torax AP, Pelvis AP.
Trauma raquimedular
Hombro doloroso Inmovilización cervical.
Larga data, limitación abducción o rotación interna o externa activa= - ASIA
Lesión manguito rotador.
-No puede despegar= Hombro congelado. Fractura de escafoides: Dolor en la tabaquera anatómica, dolor a la
-Trauma: Disyunción Acromico clavicular: Signo de la tecla radialización, dolor a la palpación del tubérculo escafoide, dolor
pedir Rx de hombro bilateral - comparativa.
a la pronación contrarresistencia y dolor a la compresión del
pulgar. Si el tratamiento es médico (no hay opción
Tumores oseos quirúrgica/muy lejos) el yeso debe incluir el pulgar. El
-Tumores benignos: Pocas cosas: Poca reacción periostica, poca tratamiento es quirúrgico.
infiltración,
-Tumores malignos: Muchas cosas Niño que cojea, dolor ingle o rodilla
-Tumor óseo más frecuente: Metástasis (mama, pulmón, próstata, Enfermedad de Perthes: Necrosis avascular idiopática de la cabeza
riñón) - Hueso es el 3 sitio de metástasis femoral, niños 6 años aprox, niños chicos, bajo peso al nacer,
Primario más frecuente: o Beningo: Osteocondroma: Cachito cerca bajo NSE, etc.
de la fisis re la rodilla o de la muñeca. o Maligno: Epifisiolisis de la cabeza femoral: Es una epifisiolistesis (la epífisis
Osteosarcoma. esta desplazada respecto a la diáfisis). 12-13 años. CLAVE:
-Sarcoma de Ewing: 5 a 15 años, fiebre, baja de peso, dolor, etc. Obeso, hipotiroideo, hipogonadismo: Niño de UP
Resto des malas, osteosarcoma
-Osteoma osteoide: Beningo. Duele más de noche, imagen en nido,
cede el dolor con aspirina. Paciente con fractura de fémur: empieza con disnea, compromiso de
conciencia y petequias en tórax = Embolia Grasa.
Paciente con trauma de rodilla, le sacan líquido y salen gotas de
Fracturas en niño grasa= Fractura. Si hay grasa, hay hueso comprometido, no
-Fracturas en niño: De la fisis (clasificación de Salter Harris), tallo
ligamentos, músculos ni tendones.
verde, torus
Fractura de cadera
-Fractura principalmente osteoporotica. Las otras osteoporoticas son Pellizcamiento acetabular: Mujeres jóvenes (manguito rotador de la
vertebral (más frecuente), húmero proximal, muñeca. cadera). Paciente con coxalgia aguda no traumática +consumo
-Posición IMPÚDICA: Muestra todo: Rotación externa, abducción, de corticoides u OH= Necrosis avascular. FR atraumáticos: OH,
acortamiento. corticoides, coagulopatía, etc
-Cirugía siempre. Pretrocanterica OTS. Medial= PTC.
-Se opera porque tiene irrigación retrógrada y terminal, porque va a
ocurrir necrosis avascular, igual que en escafoides, astrágalo y Caída de PIE: Fracturas más frecuentes: Columna y calcáneo.
cabeza del humero.)
Talalgia: Espolón calcáneo no reviste ningún significado patológico,
Fractura de muñeca no se correlaciona con fasceitis plantar. No requiere otro
-Mecanismo: Con muñeca extendida=fractura de colles (muñeca en estudio. Solo talonera.
flexion, fractura de smith) - Dorsalización, radialización,
Impactación. Posición púdica: rotación interna, aducida, acortada. Luxación
-Con muñeca flectada, fractura de Smith. Existen índices frontal, posterior de cadera (90%). Mecanismo distinto al de Fx de
sagital y varianza ulnar. No saberlos. cadera (no necesariamente viejos, si muy alta energía), Puede
Fractura de pelvis lesionar el ciático Luxación anterior se pitea el femoral
Primera opción de tratamiento: Sábana. Si ésta no sale, tutores
externos(estabilizar!). Como no sabemos que está sangrando y
TRAUMATOLOGIA
lo más probable es que sea venoso (90%), no podemos Sd atrapamiento subacromial (manguito rotador): típico en mujeres,
embolizar. La fractura tipo C es la que mata. Se debe siempre les duele hacer cosas. Todos los movimientos están
realizar TR. - Rx inlet, outlet, AP de pelvis conservados a la movilización pasiva, pero duele la abducción
contrarresistencia

Fractura por estrés de metatarsos: Fractura por desgaste. Historia


típica. Paciente corre una maraton, dolor comienza a los 10 km. Sd túnel carpiano: Durkan es signo más sensible.
A los 2 días sale a correr y el dolor empieza a los 3 km. A los 3 Sd compartimental demora horas en aparecer, si el compromiso es
dias sale a correr y el dolor empieza casi al tiro. Puede no verse inmediato pensar en lesión vascular.
Síndrome de la banda ileotibial: Dolor por cara lateral de la rodilla,
la fractura hasta la 2-3 semana.
empeora con ejercicio. Sonido al caminar o al correr
Sinovitis transitoria: Inflamación de la cadera y 2do lugar de rodilla,
Fractura expuesta post infecciosa viral, suele ser en un niño entre 2-4 años
-Factor pronóstico más importante: Compromiso de partes blandas. Enfermedad de Perthes: Necrosis avascular de la cabeza femoral,
-No olvidar VAT, ATB para gram +, gram – y anaerobios si muy niños de 5-10 años, asocia a talla baja y deporte
cochino. Penicilina si contaminada con mucha tierra. Epifisiolisis de la cabeza femoral: de 12-15 años, asocia sobrepeso,
Lesiones nervios perifericos obesidad, deporte
-Inervación del nervio mediano: Sensibilidad dedo 1 2 y 3 por volar. Y
TRAUMATOLOGIA

flexion y abducción
-Fractura de humero diafisiaria: Lesion nervio radial. No siente la
mano por dorsal y no puede extender muñeca.
-Luxación de hombro anterior: Lesion de nervio axilar: Compromete Jorge Bowakim Anta
sensibilidad región deltoidea.

3
EUNACOM RESUMEN TRAUMATOLOGIA
Traumatología es una asignatura de importancia media de cara al • Lesión de Lisfranc: consiste en una fractura-luxación
MIR. En los últimos años han formulado 7 preguntas por tarsometatarsiana. Se produce por fuerzas rotacionales sobre el
examen. Los conceptos preguntados son muy similares todos mediopie fijo.
los años y es tremendamente rentable estudiar las preguntas • Lesión de Chopart: luxación del calcáneo y el astrágalo
de los exámenes MIR previos. Desglosados por materias los respecto al cuboides y escafoides. Mecanismo de producción
temas más preguntados son fracturas y luxaciones y ortopedia similar al Lisfranc con las fuerzas rotacionales actuando a nivel
infantil, donde la dificultad de las preguntas está aumentando más posterior.
ligeramente. Del resto del temario se suelen preguntar • Fractura de Maissoneuve: lesión del ligamento deltoideo con
conceptos muy generales, por lo que si andas justo de tiempo fractura del cuello del peroné. El mecanismo es una eversión
en la preparación puede ser suficiente ampliar el contexto de con valgo forzado de tobillo transmitiendo las fuerzas
las preguntas de los últimos años. torsionales a través de la sindesmosis tibioperonea.
En las fracturas articulares está indicada la osteosíntesis si • Lesión de Bankart: consite en una avulsión del labrum
presentan un escalón articular > 2 mm. glenoideo anteroinferior. Es la causa más frecuente de luxación
Las fracturas de pelvis se observan en el contexto de anterior recidivante de hombro.
politraumatizados. En el caso de ser fracturas inestables van a • Lesión de Hill-Sachs: lesiones por impactación de la cabeza
suponer un compromiso de hemodinámica por sangrado humeral en el contexto de luxaciones de hombro.
Un apartado que hay que resaltar dentro de las fracturas es el
retroperitoneal y requieren estabilización precoz mediante
manejo de las fracturas de húmero proximal:
fijación externa. Suelen asociar lesiones vesicales y uretrales.
• Tratamiento conservador:
Un tema muy preguntado es las fracturas de cadera. Hay dos grupos
– Fracturas no desplazadas.
principales: – Ancianos con mal estado general.
• Fracturas macizo trocántereo. Se tratan mediante enclavado • Osteosíntesis:
intramedular o dispo-sitivo clavo-placa independientemente de – Fracturas de 2 y 3 fragmentos.
la edad del paciente. – Fracturas de 4 fragmentos en pacientes < 50 años.•
• Fracturas intracapsulares. Comprometen la vascularización de Artroplastia:
la cabeza femoral.Su tratamiento depende de la edad del – Fracturas de 4 fragmentos en pacientes > 50 años.
paciente y de la clasificación de Garden (Tablas 20.2 y 20.3). – Fracturas de 3 fragmentos asociadas a luxación.
Tabla 20.2. Clasificación de Garden. – Fracturas por impactación con afectación > 40% superficie
Garden I Fractura incompleta no desplazada impactada en valgo. articular (indica-ción relativa en función de la edad del
Garden II Fractura completa no desplazada. paciente).
Garden III Fractura completa desplazada en varo. 20.2. COMPLICACIONES DE LAS FRACTURAS
Garden IVFractura completa con migración proximal trocánter Globalmente siempre son más frecuentes en la tibia. Las principales
mayor. complicaciones son: pseudoartrosis, síndrome compartimental,
En las fracturas Garden III y IV se lesiona la vascularización de la distrofia simpático-refleja y necrosis avascular.
cabeza femoral, por lo que la osteosíntesis frecuentemente La pseudoartrosis puede ser:
termina en pseudoartrosis y necrosis avascular. • Atrófica: falta de aporte vascular.
Tabla 20.3. Algoritmo terapéutico. • Hipertrófica: falta de estabilidad ósea.
Garden I y II Osteosíntesis con tornillos canulados. El síndrome compartimental se produce cuando aumenta la presión
Garden III y IV < 60 años: osteosíntesis con tornillos canulados de un compartimento muscular hasta obstruir la circulación
(gran riesgo necrosis y pseudoartrosis). capilar. Es muy frecuente en fracturas de tibia, antebrazo y
60-75 años: prótesis total de cadera. supracondíleas de húmero en niños. La clínica inicial es dolor
> 75 años: prótesis parcial de cadera bipolar.
intenso en reposo que aumenta con el estiramiento muscular
Las luxaciones de cadera son otro aspecto muy preguntado en el
pasivo; si progresa aparecen trastornos sensitivos por
MIR. Hay dos tipos:
• Luxación posterior. Es el tipo más frecuente. El mecanismo compresión nerviosa y en último término alteraciones
lesional suele ser unaccidente de tráfico con traumatismo de la vasculares. El tratamiento requiere fasciotomía urgente. La
rodilla contra el salpicadero. La actitud clínica es la del bañista medida de la presión intracompartimental da el diagnóstivo:
sorprendido: miembro acortado en rotación interna y adducción PIC > (TAS-TAD)/2. (TAS = tensión arterial sistólica, TAD =
con flexión de cadera. Asociación frecuente a lesión del nervio tensión arterial diastólica). En caso de no realizar la
ciático. descompresión a tiempo, el músculo puede necrosarse y a nivel
• Luxación anterior. Menos frecuente. El miembro está alargado y sistémico se puede llegar a producir una insuficiencia renal
en rotación ex-terna. aguda por rabdomiolisis. En fase crónica de secuelas podemos
Las fracturas abiertas se clasifican, según Gustilo (Tabla 20.4): observar contracturas fijas en flexión; una de las más típicas es
Tabla 20.4. Clasificación de Gustilo. la contractura isquémica de Volkman en el síndrome
I < 1 cm, mecanismo de dentro hacia fuera. compartimental anterior del antebrazo.
II 1-10 cm. La distrofia simpático-refleja se debe a una hiperactividad del
IIIA > 10 cm. Es posible la cobertura ósea.
sistema simpático en respuesta a una lesión tisular. Fases:
IIIB > 10 cm. No es posible la cobertura ósea.
• Aguda: < 3 meses. Cambios inflamatorios con Rx normal.
IIIC Independientemente del tamaño con lesión arterial o nerviosa.
• Distrófica: 3-6 meses. Alteraciones vasculares y contracturas
Hay que reseñar que las fracturas por arma de fuego se consideran
fijas. En Rx osteo-porosis parcheada.
de entrada grado IIIA. El tratamiento de las fracturas abiertas
• Atrófica: 6-12 meses. Atrofia musculocutánea. En Rx
TRAUMATOLOGIA
hasta el grado IIIA inclusive (hueso cubierto) es como si fuera
osteoporosis difusa.
una fractura cerrada. Hay que tener en cuenta que la El tratamiento de la distrofia simpático–refleja se basa en la
discriminación entre los grados de la fractura abierta se hace rehabilitación y fisioterapia.
después de un lavado y desbridamiento amplio que siempre La necrosis avascular viene determinada por lesión del aporte
debe ser el punto inicial del tratamiento. Los grados IIIB y IIIC vascular óseo por el trazo de fractura. Es frecuente en cabeza
requieren estabilización con fijación externa para el manejo de femoral, escafoides y astrágalo.
las lesiones asociadas. En el grado I hay que asociar Recordar que también se observan casos de necrosis avascular de
tratamiento antibiótico con cefalosporinas, y en los grados II y cadera secundarios a tratamiento prolongado con corticoides y
III, cefalosporinas + aminoglucósidos. hábito enólico.
Otro aspecto que hay que considerar son las lesiones con nombre Una complicación muy preguntada es el síndrome de embolia grasa.
propio que aparecen con frecuencia en el MIR. Las más Se observa principalmente en fracturas de huesos largos no
importantes para que tengas presente son: estabilizadas (fémur típicamente) y cursa con alteración
• Fractura-luxación de Monteggia : consiste en una fractura de fluctuante del nivel de conciencia, disnea, taquicardia,
cúbito con luxación proximal del radio. petequias en tórax, cara y conjuntivas, PaO2 baja y Rx de tórax
• Fractura-luxación de Galeazzi: en este caso se fractura el radio en “tormenta de nieve” (infiltrados alveolares difusos).
y se luxa la articulación radiocubital distal. El tratamiento de urgencia requiere corticoides a dosis altas y
• Fractura de Colles: metafisaria de radio distal con soporte cardiorrespiratorio.
TRAUMATOLOGIA

desplazamiento dorsal y radial. Recuerda que se asocia con Cambiando de apartado, otro concepto reiteradamente preguntado
cierta frecuencia a lesiones del tendón extensor largo del es la asociación de la luxación posterior de hombro en
pulgar. pacientes epilépticos y descargas eléctricas.
Un detalle interesante que hay que recordar es el hematoma de
Hennequin en las fracturas de húmero proximal, que aparece a
4
EUNACOM RESUMEN TRAUMATOLOGIA
las 24 horas y discurre por cara medial del brazo y pared 20.5. LESIONES DE PARTES BLANDAS
externa del tórax. Es un tema muy heterogéneo y poco preguntado. Lo más importante
20.2.1. Lesiones nerviosas asociadas a fracturas y luxaciones son las lesiones traumáticas de rodilla.
Hay algunas muy típicas que ayudan a contestar muchas preguntas
y es imprescindible conocer (

Tabla 20.5. Fracturas y luxaciones con lesiones nerviosas típicas.


Luxación posterior de cadera. Nervio ciático. Lesiones traumáticas de partes blandas de la rodilla
Fractura de radio distal. Nervio mediano. Tabla 20.7.
Fractura de cuello de peroné. Nervio ciático poplíteo externo. Meniscos Lig. colaterales LCA LCP
Luxación anterior de hombro. Nervio musculocutáneo. Derrame Seroinflamatorio. Hemático sin grasa si lesión.
Fractura oblicua larga húmero distal. Nervio radial. CPMI. No. Hemático sin restos de grasa. No.
Fractura de epitróclea y/o cúbito proximal. Nervio cubital.
La presencia de hemartros con restos de grasa implica lesión ósea.
ORTOPEDIA INFANTIL
Dentro de este apartado es fundamental dominar la patología de la Otro concepto que hay que repasar es la lesión de Stener:
cadera del niño y adolescente. El siguiente punto más rentable interposición del tendón del adductor entre las fibras del
es el tema de las fracturas de fémur. ligamento colateral cubital del primer dedo de la mano
Otros aspectos menos preguntados son la patología congénita del impidiendo su ciactrización. Si aparece en un caso clínico un
pie, la pronación dolorosa y las parálisis obstétricas. paciente con una tumoración en el dorso de la mano que varía
20.3.1. Cadera infantil y del adolescente de tamaño a lo largo del tiempo el diagnóstico iría enfocado
Tabla 20.6. Patología de cadera infantil. hacia un ganglión.
Luxación congénita Artritis séptica Sinovitis Hay que recordar que la principal causa de hombro doloroso es el
transitoria Perthes Epifisiolisis femoral proximal síndrome subacromial. Dentro del síndrome subacromial la
EdadRecién nacido. Lactante. 2-8 años. 4-10 años. lesión más frecuentemente observada es la tendinitis del
Adolescente. supraespinoso.
F. riesgo Sexo femenino. En el hueco poplíteo se encuentra una bursa relacionada con el
Oligohidroamnios. semimembranoso denominada quiste de Baker. Es frecuente su
Macrosomía. Infección de cualquier foco. Infección
inflamación en sobrecargas mecánicas. En caso de rotura hay
respiratoria. Sexo masculino. Alteraciones coagulación.
que establecer un diagnóstico diferencial con trombosis venosa
Sexo masculino. Obesidad. Trastornos endocrinos.
profunda.
Clínica Asimetría
de pliegues.
Grados de lesión ligamentosa (esguince)
Ortolani y Barlow. Fiebre y flexo de cadera. Flexo de cadera.
Limitación rotación y abducción. Dolor a la rotación III. Distensión. Dolor en el trayecto ligamentoso que aumenta con
interna con flexión de la cadera. maniobras de estrés sin objetivar inestabilidad articular.
Diagnóstico Eco. Rx a partir 6 meses. Eco y
Cambios inflamatorios leves. Tratamiento funcional precoz y
analíticas Eco. Rx. Rx.
Tratamiento Ortesis +/– tenotomías +/– osteotomías. vendaje.
III. Rotura parcial. Aparece bostezo articular con las maniobras de
Artrotomía + AB i.v. AINES. Ortesis.
Osteotomías. Epifisiodesis. Osteotomías. estrés. Inflamación moderada-severa. Requiere inmovilización
20.3.2. Fracturas de fémur con férula un tiempo variable entre 3-6 semanas. Se puede
• < 2 años: tracción al cénit. plantear sutura quirúrgica en deportistas de elite.
• 2-6 años: tracción 90-90. III. Rotura total. Luxación articular con maniobras de estrés. Daño
• 6-12 años: tracción 90-90 vs fijación externa (dependiendo del asociado de ligamento opuesto. Requiere reconstrucción
tamaño del niño). quirúrgica mediante plastia tendinosa.
• > 12 años: enclavado endomedular elástico. frecuentemente la clínica inicial del esguince de tobillo grado I: dolor
20.3.3. Pie zambo brusco que cede de forma inicial y empeora al cabo de unas
Es una deformidad en equino, varo y adducto. El tratamiento de horas adquiriendo carácter continuo.
elección es la corrección con yesos progresivos según el MÉDULA ESPINAL Y SISTEMA NERVIOSO PERIFÉRICO
método Ponsetti +/– tenotomía percutánea de Aquiles. En este apartado lo principal es saber la clínica de las hernias
20.3.4. Pronación dolorosa discales, sobre todo las de L4-L5 y L5-S1:
Se luxa la cabeza del radio por tracción longitudinal. Se trata con un Hernia L4-L5: déficit flexión dorsal del pie. Hipoestesia cara externa
“corte de mangas”: supinación + flexión. de la pierna e interna del pie. Abolición del reflejo tibial
20.3.5. Parálisis obstétricas del plexo braquial posterior y disminución del aquíleo.
• Duchenne-Erb: C5-C6. Distocia de hombro. Antebrazo en Hernia L5-S1: déficit flexión plantar del pie. Hipoestesia cara
pronación y muñeca enflexión (“propina de maitre”). posterolateral del miembro inferior afecto. Abolición del reflejo
• Dejerine-Klumpke: C8-D1. Parto de nalgas. Mano caída y dedos
aquíleo.
separados.
20.4. TUMORES ÓSEOS Síndrome del túnel carpiano: compresión del nervio mediano por el
Lo más importante es conocer su localización y grupo de edad típico.
retináculo flexor. Cursa con dolor y parestesias en los tres
TRAUMATOLOGIA
Epífisis:
• Niños: condroblastoma. primeros dedos de la mano de predominio nocturno. Es más
• Adultos: células gigantes. frecuente en mujeres en la quinta década de la vida y en
Metáfisis: personas que trabajan muchas horas con ordenadores.
• Benigno: osteocondroma. Reproducción de los síntomas con la maniobra de Phalen. Los
• Maligno: condrosarcoma, osteosarcoma. casos persistentes o aquéllos que cursan con alteraciones EMG
Diáfisis:
del nervio mediano requieren tratamiento quirúrgico para
• < 20 años: Ewing.
• 20-40 años: granuloma eosinófilo. descomprimirlo cortando el ligamento volar anterior.
• > 50 años: metástasis, mieloma.
20.4.1. Datos típicos 1. Meralgia parestésica: compresión del nervio femorocutáneo
• Osteoma osteoide: dolor que mejora con AINES. Imagen Rx → entre el ligamento inguinal y la espina ilíaca anterosuperior. Se
nidus. presenta principalmente en pacientes obesos y embarazadas.
• Adamantimoma: insufla en “pompas de jabón” la diáfisis tibial. 2. Síndrome del canal de Guyon: compresión del nervio cubital a
• Metástasis de mama: osteolíticas. nivel de la muñeca entre el pisiforme y el ligamento volar
• Metástasis de próstata: osteoblásticas. transverso.
• Osteosarcoma: metástasis en pulmón, imagen en “sol 3. Síndrome de la encrucijada cervicotorácica: compresión del
naciente”, triángulo de Cod-man. plexo braquial y de los vasos subclavios a nivel de la axila. Es
• La presencia de calcificaciones orienta hacia un tumor de típica la reproducción de los síntomas con el test de
TRAUMATOLOGIA

estirpe cartilaginosa. hiperabducción de Wright en las compresiones


• Benignos: lesiones bien delimitadas que insuflan la cortical costoclaviculares.
respetando el pe-riostio. 4. Neuroma de Morton: compresión del nervio interdigital entre las
• Malignos: lesiones mal definidas, irregulares, que destruyen el cabezas de los metatarsianos y el ligamento intermetatarsiano.
periostio.
5
EUNACOM RESUMEN TRAUMATOLOGIA
Más frecuente en 3er espacio interdigital. Cursa con dolor • Estadío IV: artrosis establecida.
urente durante la marcha, sensación de quemazón e irradiación Tratamiento:
distal a dedos colindantes. Tratamiento inicial con plantillas y • Estadíos iniciales (I y II): descompresión central (forage)
AINES. Cirugía en casos refractarios. +/- injerto óseo.
COLUMNA • Estadíos avanzados (III y IV): artroplastia de cadera.
Hay que saber que la única indicación de cirugía urgente en
Contraindicaciones artroplastia:
traumatismos vertebrales es la presencia de clínica neurológica
• Artropatías neuropáticas.
progresiva. Es necesario conocer la asociación de artritis
• Infección activa.
reumatoide y síndrome de Down con la inestabilidad
atloaxoidea. Respecto a la escoliosis el tratamiento tiene por
Tratamiento de la artrosis
objetivo la presencia de curvas menores a 45° al final del
Inicial: analgésicos, AINES y medidas higiénico-dietéticas.
crecimiento. Se emplean diversas ortesis según el tipo de
curva, y si hay progresión se indica la cirugía basándose en dos
Alteraciones eje mecánico en pacientes < 60 años con
factores: degeneración articular parcial: valorar osteotomías
• Ángulo de deformidad de la curva. correctoras. Cambios artrósicos establecidos
• Potencial de crecimiento residual según el índice de Risser. secundarios a alteraciones eje mecánico y/o sobrepeso
Conviene conocer los conceptos de espondilolisis y espondilolistesis.
en pacientes > 65 años: artroplastia. Artrosis
Espondilolisis: lesión a nivel de la pars interarticularis. Puede
deberse a secuelas traumáticas o a defectos del desarrollo. postraumáticas en muñeca y mano de pacientes
Espondilolistesis: desplazamiento anterior de una vértebra jóvenes con altas solicitudes mecánicas: artrodesis.
sobre otra. Hay diversos tipos:
• Grado I: < 25%. POLITRAUMATIZADOS
• Grado II: 25-50 %. Su manejo debe guiarse siempre por el protocolo de
• Grado III: 50-75%. atención ABCDE:
• Grado IV: 75-100%.
• A (airway): asegurar la vía aérea.
• Grado V: espondiloptosis → el cuerpo vertebral cae sobre la
• B (breathing): comprobar y garantizar la ventilación
vértebra inferior.
Las fracturas típicas de la columna cervical son: pulmonar.
• Fractura de J efferson: estallido del atlas. Tratamiento de • C (circulation): estabilización hemodinámica.
elección con halochaleco. • D (disability): atender los trastornos neurológicos.
• Fractura de Hangman (ahorcado): pars interarticularis de C2. • E (extremities): por último ocuparse de las patologías
• Fractura de la apófisis odontoides: de los miembros.
– Tipo I: punta. Tratamiento con ortesis.
No pierdas de vista este protocolo porque las preguntas MIR
– Tipo II: base. En fracturas no desplazadas el tratamiento se
realiza con halo-chaleco. Las desplazadas se sintetizan con un sobre el manejo de los politraumas siempre han hecho
tornillo (mejor movilidad, más posibilidad de no unión), o se referencia al mismo. Hay que seguir estrictamente su
tratan mediante artrodesis C1-C2 (menos movilidad, más orden independientemente de que presente lesiones
estabilidad). muy llamativas (por ejemplo: una carótida de la que
sale un chorro de sangre que llega hasta el techo es
El síndrome del latigazo cervical se produce por un desplazamiento
brusco en flexión y extensión de la columna cervical. Es típico menos urgente que una obstrucción de la vía aérea
de los accidentes de tráfico con alcance posterior del vehículo.
Cursa con dolor cervical generalmente irradiado a columna
dorsal

alta y miembros superiores, contractura paravertebral y de ambos


trapecios, y posibles parestesias principalmente en territorio del
nervio cubital. El tratamiento inicial lo constituyen AINES,
relajantes musculares y calor local. En los casos que aparezca
una rectificación cifótica de la columna cervical en la Rx es
necesario colocar un collarín cervical y tras la fase aguda
realizar un estudio dinámico de estabilidad.

ORTOPEDIA DEL ADULTO


Necrosis avascular de cadera
Causas: traumatismos con lesión arco vascular cuello
TRAUMATOLOGIA
femoral, trastornos circulatorios (anemia falciforme,
vasculitis), radioterapia local, corticoterapia y
alcoholismo. Clasificación de Ficat (según hallazgos Rx):
• Estadío I: Rx normal. Diagnóstico por RMN.
• Estadío II: remodelación. Rx con lesiones quísticas y
áreas de esclerosis.
• Estadío III: aplanamiento de la superficie articular.

TIPOS
Alta energía
Local Neoplasia, lesiones pseudotumorales
Patológicas
Sistémica Osteoporosis (+ típica), osteomalacia, Paget…
Sobre hueso
Looser-Milkman
Estrés/ fatiga patológico
Sobre hueso sano Recluta en 2º metatarso
TRAUMATOLOGIA

TIPOS

6
EUNACOM RESUMEN TRAUMATOLOGIA
Direct
a Si reducción anatómica y no movimiento
Cortic Paso de conos perforantes en las zonas de no contacto
al Sin tejido cartilaginoso ni callo de fractura.

Formación
del Plaquetas, FC, IL 1 y 6
hematoma
Duro (O2: colágeno I y osteoblastos): osificación
Formación endomembranosa
Indire
del callo Blando (no O2: colágeno II y condroblastos): osificación
cta
endocondral

Osificació
Condroclastos y osteoclastos digieren cartílago calcificado
n
Osteoblastos depositan hueso nuevo.
del callo
Remodela Hueso fibrilar (presente en huesos fetales y callo de fractura)
ción  hueso maduro (organización según ley de Wolff)

FACTORES DE CONSOLIDACIÓN
Endocrinos: GH, insulina, anabolizantes, HT, calcitonina
Nutrición: vitamina A y D
Promueve
Medio: factores de crecimiento (IL-1, IL-6, BMPs, TGF-beta, FGFs, IGF, PDGF)
n Factores físicos: ejercicio y carga controlados, campos electromagnéticos, ultrasonidos de baja frc, O 2
hiperbárico
Endocrinos: DM, déficit GH, CC Indometacina
Malnutrición Tabaco
Dificultan Medio: hipoxia local, ↓ vascularización, perturbación del hematoma, lesión de partes Citostáticos
blandas, anemia, edad avanzada Infección
Factores físicos: distracción, compresión, cizallamiento, interposición, RTP Denervación

Vascularización precaria: cabeza húmero, escafoides, cabeza y cuello femoral, cuerpo astrágalo y tibia distal

TIPOS Osteogénico Osteoinductor Osteoconductor


Autoinjerto
Aloinjerto Solo si esponjoso (leve)
Matriz ósea
Sustitutivo óseo Cerámicas, biovidrios
desmineralizada, BMP

INDICACIONES DE CX
Politraumatizado Lesión visceral (con o sin fracturas): CX en las primeras horas
Polifracturado ≥2 fracturas sin lesión visceral: CX primeros días
Lesión vascular o nerviosa (que requiera
Por las lesiones Codo/rodilla/hombro flotante
reparación)
asociadas Abierta
Sd. compartimental
FX que necesitan reducción anatómica y
Por la naturaleza movilización precoz
FX patológica asociada a neoplasia
FX sometidas a distracción o cizallamiento
de la fractura Fracaso del TX conservador
permanente
TRAUMATOLOGIA
Necesidad de reincorporar al paciente
TRAUMATOLOGIA

7
EUNACOM RESUMEN TRAUMATOLOGIA
TX Principales indicaciones
Abstención terapéutica,
Costal aislada, algunas por fatiga
reposo
Sindactilización Dedos
Conserva Férulas, ortesis, corsés, Vertebrales, húmero, muñeca y tibia
yesos Gran parte en niños
dor
Tracción cutánea (blanda) Inmovilización provisional de cadera hasta CX
Fémur en niño
Tracción esquelética Acetábulo no desplazadas
Inmovilización provisional de fémur en adulto.
Fijación de pequeños fragmentos
Agujas
Infantiles
Fijación Cerclaje,
Rótula, olécranon
interna obenque
Osteosínt Tornillos y Antebrazo del adulto, húmero proximal y distal
QX esis placas Otras diáfisis y metáfisis con extensión intraarticular
Estabilizaci Fémur y tibia del adulto
Clavo
ón Húmero
Abiertas
Compromiso vascular Ausencia de consolidación en infectadas
Fijación externa Pelvis inestables Quemados
Intraarticulares complejas (pilón Alargamiento óseo
tibial)

COMPLICACIONES
I <1cm, limpia (por el hueso roto) TX = cerradas
Fracturas II >1cm, contaminación moderada Desbridamiento + ATB IV (+
abiertas A imp) + antitetánica +
Gustilo y I B >10cm + necesita colgajos/injertos fijador EXTERNO
Anderson I Contaminación Amputación: edad avanzada +
Tibia + FRC I C alta + lesión vascular hipoTA grave + lesiones EEII
+ >6-12h
+ FRC: tibia, antebrazo, Secuelas
codo ¡EMERGENCIA!
Sd. Dolor ++++ Retirar vendaje/yeso +
Retracciones: Volkmann (antebrazo)
compartimenta Presión IC: >30-40 elevación
Necrosis con sobreinfección
l Sí pulso (afecta vaso Fasciotomía: si persiste
IRA (mioglobinuria)
pequeño), pero débil
Alt. sensitivas y motoras
Escafoides Extirpación o injerto
Ortesis de descarga o artrodesis
Cuerpo astrágalo
Necrosis Dolor de tobillo
avascular Cabeza femoral: + RX: ↑ densidad y fragmentación
frc Prótesis
Cabeza humeral
Hiperactividad Aguda
Rubefacción, DOLOR, hiperhidrosis
SNS tras 0-3m
lesión Distróf Fría, amoratada, contracturas,
tisular ica edema, DOLOR
♀, 40-50a, 3-6m OSTEOPOROSIS moteada
muñeca- RHB
TRAUMATOLOGIA
SDRC
mano Fármacos: AINEs, CC, BB,
Distrofia
Limitación guanetidina, calcitonina,
simpaticorrefleja
funcional bloqueos simpáticos,
Sudeck Atrófic + ↓ vello, uñas y pliegues, atrofia
progresiva simpatectomía…
a muscular
ETX:
6-12m Osteoporosis difusa
inmovilizaci
ón
prolongada,
idiopática
Sd. embolia FX no estabilizadas SDRA En UCI:
grasa Huesos largos: Alt. fluctuante de O2 ± VM
diáfisis femoral conciencia: intervalo CC IV DE
+ frc lúcido 24-36h Estabilización precoz: TX y
Petequias: tórax superior, PFLX
cara, conjuntivas
TRAUMATOLOGIA

RX: torbellino de nieve


A/S: hipoxemia

8
EUNACOM RESUMEN TRAUMATOLOGIA

Diáfisis humeral
Mala vascularización
Atrófica Refrescar + osteosíntesis
Pseudoartrosis “Hoja de sable”: extremos adelgazados y
bordes
No consolidación afilados
+ + estabilización
>6m Diáfisis tibial
Hipertrófica Excesiva movilidad Injerto (placa + tornillos,
“Pata de elefante”: extremos ensanchados clavos)

TRAUMATOLOGIA
TRAUMATOLOGIA

9
EUNACOM RESUMEN TRAUMATOLOGIA
EESS

Caídas sobre extremidad Poco 2-3s Velpeau/cabestr


1/3 medio + frc, fragmento interno hacia arriba, externo desplazada (niños) illo
Clavícula
hacia abajo-adelante Desplazada 4-5s
Vendaje en 8
Fx + frc del RN (+ frc) (adultos)
Ortopédico
Cuerpo + frc, asocian fx costales y neumotórax
Escápula CX: si desplazadas o causan inestabilidad del
Alta energía
hombro
No desplazada Ortopédico
Osteosíntesis
Jóvenes
Proxi Lesión axilar o circunflejo (tornillos
Desplaz
mal Hematoma Hennequin: a las 48h, no TX 2F canulados)
ada Ancian
3-4
os Artroplastia
F
Húmero

Neuroapraxia del radial: flexión dorsal de muñeca y Espiroideas/oblicuas: + Yeso colgante


Diáfisi frc de Caldwell
mano, habitual en oblicua del 1/3 distal (Holstein-
s
Lewis), autoresolución 3-4m Transversas Clavo IM (Rush)
Supracondíleas Osteosíntesis + RHB precoz
Osteosíntesis: si no es posible extirpación y
Cóndilo + frc en niños
Distal movilización precoz
¡Aquí solo hablamos de adultos!
Epicóndilo y Conservador: CX si hay luxación y la epitróclea
epitroclea se interpone
Complicaciones: limitación flexoextensión y
pronosupinación, inestabilidad codo o longitudinal del
antebrazo
Cabeza Caída sobre palma de la mano
I No desplazada Conservador 1-2s
radio
II Desplazada + reconstruible (2 fragmentos) Osteosíntesis
Conminuta (no reconstruible)
II Anciano: resección cabeza
IV (Essex-Lopresti): + luxación RCD + lesión MIO 
I Jóvenes: prótesis
migración proximal del radio  dolor
Transversales
Osteosíntesis tipo cerclaje/tirante
Olécranon Complicación: molestias del material > extensión
(Obenque)
limitada, osificación heterotópica, neuritis del cubital
Cúbito Por protección de un bastonazo Yeso
Cúbito + radio Es inestable Osteosíntesis
FX cúbito proximal + LX cabeza radial
Diáfisis
Monteggia (proximal)
antebrazo
Lesión interóseo posterior (radial) Osteosíntesis de fractura
FX radio distal + LX cabeza cubital
Galeazzi
(distal)
Caída sobre mano en extensión
Complicación: CONSOLIDACIÓN VICIOSA  lesión
extensor largo del pulgar, distrofia simpaticorrefleja,
Radio lesión mediano, artrosis, túnel carpo
Desviación en “SUDOR” (SUpinación, Yeso máximo 6s + RX de control al inicio y
distal
Colles DOrsal, Radial) o “en dorso de semanales
tenedor” Si inestabilidad: osteosíntesis ± sustitutivo óseo
♀ post-M
Smith (Colles Desviación palmar, “en pala de
osteoporótic
TRAUMATOLOGIA
invertido) jardinero”
as Osteosíntesis
Fractura dorsal o palmar + luxación
Rhea-Barton
carpo
Hutchinson o
Estiloides radial CX si desplazadas
del chófer
Yeso 2-3m (incluir 1º dedo)
Jóven + caída sobre talón de la mano forzando extensión Si CLX pero no se visualiza: yeso
No
de muñeca + RX en 2s para confirmar
desplazada
Escafoides Dolor + tumefacción tabaquera anatómica
o descartar
Complicaciones: pseudoartrosis, necrosis avascular (polo
Desplazad
proximal tiene peor irrigación), artrosis 2ª Osteosíntesis
a
Oblicua, SLP desplaza
Bennett Osteosíntesis
Base 1º diáfisis a proximal
MTC
Rolando Ortopédico
TRAUMATOLOGIA

ORRRtopédico
Cuello 5º
Boxeadores
MTC

10
EUNACOM RESUMEN TRAUMATOLOGIA
Falanges Sindactilia y movilización precoz

TRAUMATOLOGIA
TRAUMATOLOGIA

11
EUNACOM RESUMEN TRAUMATOLOGIA
CADERA
INTRACAPSULARES EXTRACAPSULARES
Necrosis avascular: la cabeza tiene mala
Complicaciones vascularización Consolidación en mala posición
Pseudoartrosis
General Acortamiento + dolor + impotencia funcional
RE y
CL + Borde lateral pie tocando camilla y gran abducción
abducción
X
Hematoma
- +
visible
URGENTE: <6-8h Puede demorarse más: ideal <24-48h

CX precoz

Pertrocant Tornillo deslizante


Jóvenes
Osteosíntesis Osteosínt érea (DHS)
No desplazada (tornillos canulados) esis a
>6 (I-II) cualquier Subtrocant Clavo corto de
5a Desplazada edad érea cadera (gamma)
Artroplastia
(III-IV)

EEII
Estables Avulsiones: jóvenes DESCARTAR
Conservador
1 punto FX de ramas: ancianos Hemorragias:
Pelvi hipoTA
s Alta energía Roturas vesicales
Inestables Fijador externo de urgencia 
Complicaciones + frc Rotura de uretra
>1 punto Mortalidad: 10-20% posterior osteosíntesis
Lesión ciático

Desplazadas (>2mm)
Osificación Subluxada (inestable)
Alta energía Osteosínt
Acetábulo heterotópica No reductible
Asociadas a pelvis esis
Artrosis Atrapamiento de
fragmentos
Tracción TE: en espera de
Adulto
CX, evita embolia
Embolia grasa s
Diáfisis Importante sangrado + clavo IM (Küntscher)
Pseudoartrosis
Tracción TE 90-90 2s +
Niños
Fémur 2s yeso
No
Conservador
Supracondil desplazada
Puede afectarse a. poplitea
ea Desplazada, Osteosíntesis: +
escalón frc
No Conservadora o
desplazadas patelectomía parcial
TRAUMATOLOGIA
Rótula Transversales: + frc
Desplazada 2 agujas + 1 cerclaje
(+ frc) (Obenque)
No
Lateral (valgo forzado):lesión meniscal/ligamentos Conservador
Meseta desplazada
Medial (varo forzado): lesión n. peroneo común 
tibial Desplazada,
Tibia dorsiflexión tobillo/pie, marcha estepaje Osteosíntesis
escalón
Sd. compartimental Cerradas Clavo IM
Diáfisis Localización + frc de FX abiertas
Pseudoartrosis Abiertas Fijador externo
Suprasinde
Maisonneuve: 1/3 proximal peroné + l. deltoideo
smal
Osteosíntesis
Transindes Dupuytren, fractura-luxación o BIMALEOLAR: 1/3
mal distal peroné + l. deltoideo
No desplazadas y no
Tobillo
afectado CIT (l. Conserva
TRAUMATOLOGIA

Infrasindes deltoideo, maléolo dor


mal interno)
Osteosínt
Resto
esis

12
EUNACOM RESUMEN TRAUMATOLOGIA
Sd. compartimental No desplazada Conservador
Artrosis Desplazada
Precipitados postraumática Avulsión Osteosíntesis
Calcáneo Asociación: FX meseta tibial Tenosinovitis inserción aquilea
Pie vertebral y calcáneo CL Lesiones Artrosis Artrodesis
almohadilla postraumática subastragalin
grasa invalidante a
Signo de Hawkins: resorción Desplazada Osteosíntesis
Astrágalo Necrosis avascular
subcondral, buen PX No desplazada Conservador
2 Cuello
Estrés o fatiga Conservador
º Recluta
MT
Metáfisis
T 5
prox. Mala vascularización: riesgo de pseudoartrosis Osteosíntesis
º
Jones
Peroneo lateral corto y fascia plantar: avulsión estiloides
Base Yeso
Tocar siempre en esguince de tobillo  RX si dolor

Elemento + importante de estabilidad del anillo pélvico: ligamentos sacroilíacos posteriores

TRAUMATOLOGIA
TRAUMATOLOGIA

13
EUNACOM RESUMEN TRAUMATOLOGIA
2. LUXA CIONES

Antes de reducir, pedir siempre RX o TC para certificar la luxación y descartar fracturas

GLENO-HUMERAL HOMBRO
TIPOS CLX CARACT. RX TX
Reducción cerrada ± fijar troquíter (si
Abducción y Rotacion Externa Lesión del N axilar
Anterior desplazado)
Movilización dolorosa (circunflejo) Visible
+ frc + vendaje Velpeau: 2-4s
Deformidad en charretera ± fractura troquíter + RHB
Posterior Pérdida de conocimiento,
Adducción y Rotacion Interna Poco
descarga eléctrica, crisis Reducción cerrada bajo AG
Electrocuta Dificultad RE y abducción
convulsivas
Puede pasar desapercibida llamativa + inmovilizar en discreta RE
dos ± fractura troquín
Perthes-
Bankart Avulsión rodete glenoideo y
CX reconstrucción
Traumáti (antero- ligamentos GH inferiores
ca inferior)
Recidivant
Fractura por impactación
e Hill-Sachs Ortopédico
posterolateral de cabeza
No
traumáti Laxitud ligamentosa RHB
ca
Pequeña: reducción abierta
Luxación desapercibida y no reducida en 1º momento: en posterior + frc Mediana: transposición de
Inveterada Ocupación fibrosa, erosión, contractura, elongación…
Se dificulta reducción cerrada y estabilidad subescapular o troquín
Grande: artroplastia

CADERA
Posterior: bañista sorprendido Anterior impudico
FRC 90% 10 %
Trauma de gran energía ± fracturas acetábulo/cabeza femoral
CLX Acortada + RI + ADucción Alargada + RE + ABDucción
Lesión ciático Lesión PVN femoral
Complicacione
Necrosis isquémica, coxartrosis
s
TX URGENTE
Reducción cerrada bajo anestesia  estabilización transitoria con tracción  carga parcial 2-3m
Evita CX si: fractura acetabular quirúrgica, fragmentos intraarticulares incarcerados o imposibilidad de reducción cerrada
complicaciones

CC íntegros
Integridad total Esguinc
Conservador
AC: no estabilidad horizontal  lesión  (I) e
desplazamiento AP del extremo distal clavícula AC rotos (II)
AC
CC: no estabilidad vertical  desciende escápula Luxació
por peso de EESS n III: CX si estética o
CC rotos (≥III)
Tecla demanda funcional
+
Caídas sobre EESS  se desplaza olecranon y
TRAUMATOLOGIA
Codo cabeza radial Ortopédico ± osteosíntesis de fracturas asociadas
Posterior + frc, ± FX de coronoides
Caídas sobre mano en hiperextensión Reducción y estabilización (agujas) + reconstrucción
Carpo Luxación de ligamentos perilunares
ligamentosa
± lesión mediano
FR: rótula alta, displasia femororrotuliana 1º episodio: ortopédico, CX si es irreductible
Rótula
♀ (2D), + frc hacia lateral, derrame Recidivante: RHB  CX si no efectivo
Alta energía, inestabilidad multidireccional URGENTE: reducción cerrada e inmovilización bajo AG +
Rodilla ± lesión a. poplítea: ECO-doppler para descartar, vigilancia con el paciente ingresado +
arteriografía si dudas ± n. peronéo reconstrucción diferida de estructuras lesionadas

Calcáneo-astragalina/cuboides/escafoides
CHOpart CHOnservador
Estables, deformidad llamativa

LisfranQX Tarso-metatarsiana
TRAUMATOLOGIA

Osteosíntesis
+ frc Inestables, posible desapercibida

14
EUNACOM RESUMEN TRAUMATOLOGIA
3. PART ES B LA NDAS

Derra
Estructura MA EF TX
me
Déficit pinza
CL cubital
Stenner: no cicatrización por
(MCF)
Separación interposición de Parcial: conservador
Guardabosque
forzada aponeurosis del Completa: CX
s
aproximador corto del
Esquiador
pulgar
Seroso CPMI + frc  dolor en IL
Giro + Diferid Estable: nada
posterior
CLX (bloqueos, derrames,
rodilla o Dolor/chasquidos con
Meniscos fallos, asa cubo):
flexionada y (18- FLEXIÓN y rotación
meniscectomía parcial,
cargada 24h) (externa el medial, interna
± sutura (periferia)
Ligero el lateral)
artrocentesis:
Hemáti
si derrame
Hiperextensi co LAChman, cajón anterior
Fractura de Segond: avulsión + conservador
LAC ón o giro Rápido CX: casi siempre
del margen tibial lateral inicial
con valgo (1-2h)
EF + RM: ya
Mucho
Translación enfriada Conservador: aislada y
Cajón posterior
LPC tibial Variable estable
Recurvatum
posterior QX: inestable, LCLL
completa, LPC +
Dolor Con varo a 30º
Lateral Varo forzado LCLL/poplíteo/bíceps/cáp
LC Inestabilida de flexión
sula PL
L d
Valgo Con valgo a 30º
Medial Abertura Conservador
forzado de flexión
Urgencia + frc: PA anterior +
lesionado
Dolor: aparece con trauma, se
va por continuar con el
movimiento, pero al
CL lateral
Inversión enfriarse vuelve constante
(tobillo) Conservador
forzada 5% secuelas: inestabilidad,
3 haces
dolor articular por lesión
cartílago
Descartar otros niveles: palpar
base 5 MTT, m. pedio y
articulación de Lisfranc

Tríada desgraciada (O’Donoghuel): MM + CL medial + CA (si la tienes te vas en CA-MI-LLI-ta)

Causa + frc de hombro doloroso


Atrapami SU SUINg REDONDito: SUpraespinoso, SUbescapular,
ento INfraespinoso, REDONDo menor Conservador  RHB  infiltraciones
subacrom Dolor mecánico: Acromioplastia: NO respuesta 3-6m o
Inicio inisidioso, predominio nocturno
TRAUMATOLOGIA
ial jóvenes con rotura aguda
Con la bajada activa del brazo
♀ >40a Neer: dolor con elevación pasiva máxima (signo),
desaparece con infiltración de anestésico (test)
EpiconDiliti
Tenis (naDal), lateral M. extensora-supinadora
s
EpiTROcleit
Golf, medial (denTRO) M. flexora-pronadora
is
Extensor corto y abductor largo del pulgar a su paso sobre la
Dequervai
estiloides radial
n
Test de Finkelstein
Conservador
Fascitis
Talalgia, espolón calcáneo, tardan meses en resolver CX: si fracaso
plantar
Ganglione
Dilataciones de sinovial herniados
s
Quiste Acúmulo de líquido en bolsa serosa común de semimembranoso
TRAUMATOLOGIA

de Baker y gemelo interno Hueco poplíteo


Olecraniana: estudiantes
Bursitis
Prerrotuliana: beata

15
EUNACOM RESUMEN TRAUMATOLOGIA
♂, 50-60a, AD con penetrancia incompleta, OH, epilepsia Fasciectomía:
BIL asimétrica, indolora, inicio y mayor intensidad en Limitada: pequeñas porciones de la
Dupuytre CUBITAL afectada
Nódulos y bandas fibrosas en fascia palmar  contractura Regional: + frc, toda la afectada
n
progresiva Radical: afectada y sana
Asociación: plantas pies (Ledderhose), pene (Peyronie, dolor), Fasciotomía PC: recidivante, solo
dorso de IFP (Garrod) ancianos con CI de CX
Rotura ♂, 30-50a, deportistas ocasionales de fin de semana
Signo de la pedrada, chasquido, impotencia para flexión plantar CX
Aquiles
Signo del hachazo, maniobra de Thompson

TX esguince extraarticular: RICE  rest, Ice, compressión, elevation


Lesión meniscal de + bloqueo e incapacidad: en asa de cubo
Gotas grasa en artrocentesis: indican fractura articular
MI forma de C, ME forma O
Bolsa cubital o vaina cubitocarpiana = vaina común de los flexores
Lesión ligamentosa + frc de rodilla: LLI

TRAUMATOLOGIA
TRAUMATOLOGIA

16
EUNACOM RESUMEN TRAUMATOLOGIA
4. SNP

CLASIFICACIÓN LESIONAL DE SEDDON


Tipos Lesión TX Recuperación
Neuroapraxia Edema 3s a 3m
Conservador
Axonotmesis Axones 1 mm/d
Neurotmesis Axones y envolturas CX Incompleta

COMPRESIONES NERVIOSAS: TX conservador  CX si fracaso


Encrucijada Compresión de plexo braquial y vasos subclavios
cervicotorácica La CX implica resección de la 1º costilla
Proxi Sd. del
Escritores
mal pronador
Neuropatía compresiva + frc
Asociación: inflamación sinovial (AR, sobrecarga), amiloidosis,
hipoTD, embarazo, gangliones, lipomas, implantación
muscular anómala, desviaciones óseas…
Parestesias en 1, 2, 3 y mitad del 4 dedo, + frc
nocturnas y mano dominante
Phalen y Tinnel, RX, EMG y velocidades de conducción
La CX implica sección del ligamento anular

Aspectos Esenciales
Es la neuropatía por compresión más frecuente.
Se asocia a tenosinovitis de los flexores del carpo,
Mediano

secundaria a movimientos repetidos.


Se presenta con dolor y parestesias en zona de
Túnel del
Distal inervación del nervio mediano.
carpo El tratamiento inicialmente es médico, indicándose
cirugía ante compromiso severo, refractario o motor.
Caso Clínico Tipo
Paciente de sexo femenino, 45 años, secretaria, con antece-
dentes de hipotiroidismo, consulta por cuadro de dolor
nocturno y parestesias en cara palmar de dedo índice,
medio y pulgar. Al examen físico Signos de Phalen y Tinel
(+).El signo de Tinel consiste en que la percusión del nervio mediano en
el túnel del carpo reproduce el dolor y las parestesias La maniobra de
Phalen consiste en realizar una flexión de la muñeca durante 1 minuto, lo
cual reproduce la sintomatología y se encuentra presente en el 75% de
los casos. El signo de Flick consiste en que el paciente sacude la mano
afectada cuando se le pregunta qué hace al sentir los síntomas.

En codo o entre los 2 vientres del flexor cubital del carpo


Parestesias mitad 4 y 5 dedo, garra cubital, atrofia
Proxi Túnel
Cubital

hipotenar
mal cubital La CX implica transposición SC o submuscular (junto al
mediano) ± epicondilectomía
Canal de
Distal
Guyón
Radial En arcada de Fröhse  dolor distal a epicóndilo
Compresión del femorocutáneo entre EIAS y ligamento inguinal  dolor y parestesias en la
cara anterolateral del muslo
Meralgia parestésica
Obesos, embarazadas, CX, ↑ presión IA (sobreesfuerzos)
TRAUMATOLOGIA
La CX implica descompresión QX
Conducto de Hunter (surfistas, corredores): dolor en cara medial de muslos y rodillas
Safeno
Rodilla: por CX
Anterior Peroneo profundo  dolor en 1º espacio interdigital
Túnel del tarso
Posterior Tibial posterior  dolor en planta del pie
♀, 40-50a
Compresión n. interdigital (+ frc entre 3-4 MTT) ± neuroma
Neuroma interdigital Dolor quemante sobre espacio afectado con la marcha y bipedestación que irradia a pulpejo de
de Morton dedos
ECO, RM
La CX implica resección del ligamento interMTT dorsal y del neuroma

El mecanismo de pinza requiere: mediano + radial


TRAUMATOLOGIA

17
EUNACOM RESUMEN TRAUMATOLOGIA

TRAUMATOLOGIA
TRAUMATOLOGIA

18
5. TUM ORES

EPIFISIS: EPI, BLAS y CLAS METÁFISIS

Céls. gigantes =
OsteoBLAStoma CondroBLAStoma Osteocondroma CondroSARCOMA OsteoSARCOMA
osteoCLAStoma
Adulto joven Infancia Adulto joven Infancia Adulto Bimodal: 10-25 y 50-60a
Calcificaciones Sol naciente
Calcificaciones,
Burbujas En sal y pimienta o palomitas de Fuego en hierba, triángulo de
algodonoso
maíz Codman
Raquis posterior Rodilla Distal fémur, tibia proximal Cintura, axial >50% rodilla
1ª + frc
Dolor NO cede con ↑ durante crecimiento: no Blástico
♀ ↑ embarazo RTP/QTP resistente
AAS ♂ TX hasta fin de este si M pulmonares: 98%
M pulmón <5% CX radical
Legrado amplio asintomático Paget, Li Fraumeni
Sd. compresión

DIÁFISIS: GEMMA estudia tOdO el día

Granuloma eosinófilo Ewing Mieloma Metástasis Adamantinoma Osteoma Osteoide


Infancia 10-15a >50a 20-50a Adulto joven
Sacabocados, vértebra Perióstica en capas de Osteoblásti Osteolíti Mixtas: +
Osteoporosis Burbujas Nidus: una O dentro de otra O
plana cebolla ca cas frc
Cráneo, axial Cráneo, raquis Tibia (90%) Femorotibial, raquis posterior
CA Osteoblástico, cortical
tiroides TC ver extensión
90%: t(11, 22) Paget
Histiocitosis de céls. Leucemia El resto Dolor nocturno que CEDE con
Radiosensible CA próstata
Langerhans s AAS
Linfomas Autorresolución en 5-7a 
+ frc en niños de todos Maligno 1ª + frc Tumor + frc ablación RF

Malignizan 2% (10% si múltiples)


Falanges de mano Cráneo
Endocondroma Calcificaciones Osteoma
20-30a 20-30a
Múltiples en: Ollier, Maffucci
Sacro y clivus Fibroma no
Cordoma Restos notocorda Desaparece en adolescencia
40-70a oseificante
Displasia Lesión benigna + frc de costillas Quiste Rodilla Hemático, dolor asociado a trauma
Burbujas
fibrosa aneurismático Metáfisis Burbujas
Niñas, Albright
Cráneo y raquis (cuerpo) Vascular Quiste óseo 7-20a
Hemangioma Curación espontánea
40-60a Curetaje o RTP esencial

BX: siempre
TX: multidisciplinar (RTP, QTP, CX)
Calcificaciones: tumores condrales
6 . INFAN TIL Y A DOLESC ENTE

FX del parto: clavícula > húmero

LESIONES PROPIAS

EPIFISIÓLISIS

Complicación + frc  epifisiodesis

I
Conserva
II Muñeca
dor
+ frc
Chaput Tillaux:
III
tibia
Tobill CX
Triplanar
o (agujas)
IV Cóndilo humeral
lateral
V
TX
+ Detención del crecimiento
secuelas
grave
En rodete, “caña de bambú” Incurvación plástica En tallo verde

Estable Inestable

CADERA: cojera, dolor, claudicación

Sinovitis
Displasia congénita Perthes Epifisiólisis
transitoria

RN Escolar (3-9a) Adolescente (11-16a)

♀ ♂

Edad

Alt. coagulabilidad
sanguínea
Nalgas, primogénitos Obesidad, alt. endocrinas
Infección VR Talla baja
FR AF, macrosomia, Retraso maduración sexual
previa Hiperactividad, tabaquismo
oligohidramnios Afroamericanos
pasivo, alt.
somatomedinas

Limitac
Abducción RI y abducción RI
ión
Agudo
Dolor
irradiado Dolor crónico sin trauma: 2/3
Barlow y Ortolani +
CLX a la Insidiosa muslo, 1/3 rodilla
Asimetría de pliegues
rodilla Miembro en RE
± derrame
seroso

ECO: si <3m RM urgente


DX De exclusión RX
RX: si >3m RX

TX Reducción cerrada Sintomático F. hipovascular: descarga Reducción cerrada


<6 Agud
m + arnés de Pavlik 2- F. hipervascular: moverse a + epifisiodesis
3m mucho (tornillos)
6m- Reducción cerrada + Cróni Epifisiodesis
1a yeso ca (tornillos) IN SITU
Reducción abierta
>1a + osteotomías fem. + CX PROFILAXIS CL
Mal PX: obesidad, tardío,
y/o acetab.
mujer, debut brusco,
CODO
Pronación dolorosa, codo de niñera Fractura supracondílea
<4-5a 6-7a

6-7a
PATOGEN Subluxación cabeza radial
Fragmento distal desplazado a posterior
IA fuera del ligamento Trauma y dolor en codo infantil  FX supracondílea hasta
anular que se demuestre lo contrario

Complicaciones
Dolor
N. interóseo (rama del mediano): + frc
Codo en semiextensión
CLX Vena braquial: fractura que causa + frc sd.
Antebrazo en pronación
Ausencia de movilidad activa de la ES compartimental en el niño
Cubital: tardía, por agujas
Sin pulsos No Exploración abierta de a.
Reducción recupera humeral
urgente Recupera
Supinación forzada de antebrazo + flexión Desplaza Agujas
TX
No necesita inmovilización posterior da
Con pulsos
No
RX
desplazad Yeso
a

PIE
Zambo Plano-valgo Cavo

Image
n

Flexible Rígido-doloroso
Corrige al levantar 1º dedo
CARAC EVA: supino-equino- Origen neurológico + frc 
(Jack/Rose) o ponerse
T. varo-aducto Coalición tarsal hacer EF + EMG
de puntillas
Asintomático
Yesos (en orden
Ortesis/yeso en
inverso: AVE)
TX Seguimiento descarga 6s Ortesis, CX si no cede
± tenotomía de
CX si fracaso
Aquiles

OSTEOCONDROSIS + RELEVANTES: centros de osificación  degeneración secuencial/necrosis aséptica + recalcificación


Freiberg (Köhler
Cabeza del 2º MTT ♀ Metatarsalgia anterior, ↑ de puntillas
II)
Epífisis femoral
König Cóndilo interno + frc
distal
Adolescente
Osgood- Tuberosidad tibial La + frc
TX conservador
Schlatter anterior Niños deportivamente activos
Preiser Escafoides carpiano
Scheuermann CCVV ♂
Infantil,
Köhler Escafoides tarsiano
escolar
Epífisis femoral
Perthes Escolar
proximal
Kienböck Semilunar Adulto joven
7. CX RECONSTR UCTI VA DEL A DU LTO

Aguda
Desbridamiento amplio + retención del implante + ATB 3m
Prótesis (<1m)
infectada Recambio en 2 tiempos: quitar prótesis, lavado QX, espaciador de cemento con ATB 
Crónica
prótesis nueva en 2-3m
Aflojamiento
Recambio directo
aséptico
Hallux valgus Osteotomía: solo si duele
Artropatía
Artrodesis
neuropática

ARTROSIS
Generalida FR: edad, ♀, obesidad
des Disociación clínico-radiológica, dolor mecánico, no RFA, posible rigidez matutina de corta duración
IFD (Hederben): >40a, asociación familiar, + frc
Gonartrosis, coxartrosis
Formas 1ª que Bouchard Vertebral
habituales IFP (Bouchard): 1 MTF (Hallux rigidus)
Trapecio metacarpiana (rizartrosis)
Pinzamiento irregular de la interlinea articular
Esclerosis subcondral
RX
Osteofitos, geodas
Deformidad articular
↓ demandas
Médi VO: paracetamol  AINE, COX-2, glucosamina, condroitina-sulfato, S-adenosilmetionina  opiáceos
co débiles (codeína, tramadol)
Infiltraciones: CC, analgésicos, hialurónico
Cadera
TX Rodilla Artropla Af. intermedia con genu varo: osteotomía tibial valguizante
Hombr stia
CX
o
Tobillo
Artrodesis
y pie

NAV: + frc OPT


EDAD 20-40a
Idiopática, OH, CC o Cushing ♀: embarazo
ETX Traumática, trasplante renal, Hbpatías, pancreatitis, sinovitis, Gaucher, gota, metabólicas, ♂ : 40-50a
RTP
Debut Insidioso Agudo
Laterali
BIL >50% UNIL
dad
Sordo, intermitente, en reposo, movilidad limitada (sbt RI) Muy intenso con la
Dolor
Ingle, nalgas y/o rodilla marcha
Patognomónico: colapso cabeza femoral, radiolucencia subcondral, secuestro
RX anterolateral Osteopenia
Esclerosis
↑ homogéneo
GMF Hipocaptación rodeada por halo de hipercaptación
captación
T1: lesión focal hipointensa, lo + precoz
RM Edema difuso
T2: doble línea (línea de hipointensa rodeando zona de hiperintensa)
I RX sin cambios Forage
Áreas quísticas reabsorción y cambios osteoescleróticos en descompresiv
II
zonas de reparación o
R Buen PX
TX II
X Colapso subcondral y aplanamiento de cabeza femoral Conservador
I Artroplastia
I total
Cambios degenerativos de artrosis 2ª
V

Causa + frc de dolor mecánico y cambios degenerativos en ♀ jóvenes: displasia de cadera


CI de artroplastia de sustitución: artropatía neuropática de Charcot e infección activa
8. COLU MNA VE RTE BRA L

Espondilolistesis: anterolistesis + frc


Ístmica: + frc Degenerativa
Nivel L5-S1 L4-L5
Pacien
Joven deportista Mujeres edad avanzada
te
ETX Espondilólisis pars interarticularis (estrés + frc) Degeneración  inestabilidad
Aplanamiento de lordosis (sin culo)
Dolor lumbar: ↑ bipedestación y marcha, desaparece
CLX sentada/tumbada Dolor lumbar
Contractura isquiotibiales Claudicación neurogénica (estenosis de canal)
FLEXIÓN DEL TRONCO LIMITADA
RX oblicuas: borra cuello del perrito
DX
TC (valorar pars), RM (si CLX neurológica), SPECT (+ S)
Ortesis y reposo: si dolor lumbar + desplazamiento
TX <50% (II) Ortesis  descompresión + artrodesis si fracaso
Artrodesis: resto

OTRAS DEFORMIDADES
RX: normal o incurvación sin
Actitud
rotación
escoliótic -
Adams normal, hiperextensión
a
corrige
<30 Observación: cada 4-6s
Escoliosis Asintomática, plan frontal
º hasta Risser 4
idiopática (torácica derecha)
RX: incurvación + rotación, Corsé: altas Milwaukee, bajas
Complicaciones: IP, cor 30-
adolescen Cobb >10º Boston
pulmonale 50º
te Adams manifiesta giba No sirve si Risser ≥4
Risser: madurez esquelética según
♀ >50
cresta ilíaca. ≥4  100% CX
º
Giba con postura normal,
Cifosis de que no corrige con
Scheuerm hiperextensión
Dorso-lumbar, plano sagital Osteocondrosis  acuñamiento
ann Corsé Milwaukee ± yesos
Dolor: ↑ bipedestación y marcha, ≥3 vértebras >5º  ↑
♂, CX: fracaso o cifosis >75º
desaparece sentado/tumbado) cifosis
adolescent Estrechamiento e irregularidad
e de discos IV, nódulos de
Schmorl
Hombres sin cuello, tortícolis
irreductible, rigidez cervical
Klippel- ♀=♂, esporádica, AD o AR Implantación baja del cabello, No TX, excepto CLX o inestabilidad
Feil Fusión de cuerpos vertebrales
anomalías faciales,
trastornos neurológicos

FRACTURAS CERVICALES RELEVANTES


I Punta Halo-chaleco
Osteosíntesis con tornillo
Fractura de odontoides II Base
Artrodesis C1-C2
III Cuerpo Tracción craneal + halo-chaleco
Espondilolistesis del I Estable y no desplazada Halo-chaleco
axis II: + Lesión discal C2-C3 + traslación y angulación
Tracción craneal + halo-chaleco
Ahorcado, hiperextensión frc de fractura
Pars interarticularis de C2 III Asocia luxación unifacetaria o bifacetaria Osteosíntesis

FRACTURAS DORSO-LUMBARES
Lesión
neurológic
Tipos a TX
(columna
media)
>50% aplastamiento
Por compresión >50% ocupación canal CX (artrodesis)
Aplastamiento/acuñami >25º acuñamiento dorsal
-
ento Corsé y reposo
Lumbar + frc Estables, osteoporóticas Vertebroplastia PC: si osteoporótica que no
controla dolor
Flexión-distracción <10% CX Afectación ligamentosa o neurológica
Estallido 50% si Déficit neurológico, cifosis >25º u
ocupación canal >50%
Fracturas-luxaciones La que + Siempre

CX urgente de fracturas: solo si lesión neurológica incompleta y progresiva

¿Metilprednisolona IV <8h en lesión medular aguda cual es la más característica: A) Pie cavo-valgo
(NASCIS)? A día de hoy ya no se da postraumático B) Necrosis isquémica del cuerpo del
Shock medular (DD shock neurogénico): ausencia de función astrágalo C) Pie plano postraumático D) Pie cavo-varo
medular debida no a lesiones morfológicas, sino a disfunción postraumático E) Sindrome compartimental
Explorar reflejo bulbocavernoso B
Impide valorar extensión y progresividad hasta que no cede
9
(normalmente <24h)
Lesión de elementos vertebrales posteriores (arco y
Una mujer de 47 años consulta por dolor en el hombro
ligamentos)  inestabilidad
derecho, de origen insidioso a lo largo de semanas. Es más
TC: mejor prueba para valorar ocupación medular por fractura intenso durante la noche y aparece con algunos
vertebral movimientos de la extremidad superior, especialmente
FX osteoporótica + frc: vertebral cuando limpia y cuando maneja. Al examen físico, el
2 hombro presenta movilidad pasiva completa e indolora,
Una mujer de 66 años sufre caida de altura, golpeándole la pero aqueja dolor a la abducción contra resistencia. El
cadera y la extremidad inferior izquierda, evolucionando con diagnóstico más probable es: A) Síndrome de atrapamiento
imposibilidad de ponerse de pie y gran dolor. Al examen subacromial (manguito rotador) B) Luxación crónica de
físico destaca extremidad inferior izquierda en rotación hombro C) Capsulitis adhesiva D) Artrosis E) Fibromialgia
interna y aducción, con algún grado de acortamiento. Los A
movimientos de la cadera están limitados por intenso dolor. 10
El diagnóstico más probable es: A) Fractura de cadera La primera medida a realizar en un paciente
intertrocantérica B) Fractura de cuello femoral C) Luxación politraumatizado es: A) Asegurar la vía aérea B) Poner una
posterior de cadera D) Fractura de pelvis E) Necrosis tabla espinal C) Detener hemorragias activas D) Administrar
avascular de la cabeza femoral cristaloides endovenosos E) Apoyar la ventilación y
C - Luxación posterior de cadera administrar oxígeno
3 A
Un hombre de 26 años sufre una caída mientras esquiaba, 11
torciéndose la rodilla derecha y evolucionando con La fractura de Colles presenta: A) Radialización, impactación
importante dolor, especialmente al caminar. El examen y desplazamiento dorsal del extremo distal del radio B)
físico se aprecia pequeño derrame articular y dolor a la Cubitalización, impactación y desplazamiento a palmar del
rotación externa de la pierna, a la compresión de la rodilla y extremo distal del radio C) Cubitalización, rotación interna y
a la palpación de la interlínea posteromedial. Las desplazamiento a dorsal del extremo distal del radio D)
radiografías de rodilla descartan lesiones óseas. ¿Qué Radialización, impactación y desplazamiento a palmar del
estructura se dañó con mayor probabilidad? A) Menisco extremo distal del radio E) Radialización, rotación interna y
medial B) Menisco lateral C) Ligamento cruzado anterior D) desplazamiento a palmar del extremo distal del radio
Ligamento cruzado posterior E) Ligamento colateral medial A
A - Menisco medial 12
4 Una paciente sufre un accidente de tránsito, resultando en
La etiología de la distrofia simpático refleja es: A) Isquemia una fractura de pelvis, desplazada e inestable. No tiene
de la vasa nervorum B) Analgesia insuficiente C) lesiones torácicas ni faciales y respira adecuadamente.
Inmovilización deficiente D) Inmovilización prolongada E) Inicialmente se encuentra en buenas condiciones, pero
Compresión nerviosa adolorido Veinte minutos después, evoluciona con
B taquicardia, palidez, frialdad e hipotensión. La conducta
5 más adecuada es: A) Reponer volumen y realizar
Al servicio de urgencias es traído un niño de 5 años por angiografia urgente con embolización de los vasos
dolor inguinal de 48 horas de evolución. Tres semanas antes sangrantes B) Administrar cristaloides endovenosos y
ha presentado infección de vías respiratorias altas tratada estabilizar la fractura con un tutor externo C) Reponer
con antibióticos. La exploración abdominal es normal y no volumen con suero fisiológico y realizar laparotimia
se palpan hernias. La cadera presenta limitación dolorosa exploradora, de modo de ligar los vasos sangrantes D)
en todos sus movimientos, y el paciente cojea. La analítica y Administrar cristaloides, coloides y glóbulos rojos hasta
la radiología son normales y la ecografía indica un pequeño lograr estabilizar la volemia E) Realizar osteosíntesis de
derrame articular de cadera probablemente seroso. El cadera y transfundir glóbulos rojos
primer diagnóstico de sospecha debe ser: A) Artritis séptica B
de cadera B) Artritis reumatoide juvenil C) Sinovitis 13
transitoria de cadera D) Enfermedad de Perthes E) El dolor en la tabaquera anatómica es un signo típico de: A)
Osteomielitis hematógena proximal de fémur Tendinitis de Quervein B) Dedo en gatillo o resorte C)
C Atrapamiento del nervio cubital D) Fractura del escafoides
6 E) Artrosis carpometacarpiana del primer dedo (rizartrosis)
Un paciente de 21 años, que ha sufrido una caída vertical D
sobre la pierna manteniendo el tobillo en varo y rotación 14
interna, ha sido diagnosticado de esguince externo del Los síntomas característicos del síndrome de túnel carpiano
tobillo derecho. ¿Cual sería la técnica exploratoria más consiste en: A) Parestesias y debilidad a la flexion de los 1,
adecuada de las que se relacionan, para evaluar la 2 y 3 dedos B) Parestesias y debilidad a la flexion de los 4 y
gravedad del cuadro? A) Una TAC de pinza bimaleolar B) 5 dedos C) Parestesias y debilidad a la extension de los 1, 2
Una RM del tobillo C) Rx dinámicas del tobillo previa y 3 dedos D) Parestesias y debilidad a la extension de los 4
anestesia local o regional D) Repetir la Rx estándar pasados y 5 dedos E) Parestesias y debilidad a la flexión de todos los
7 días E) Una cintigrafía con TC 99 dedos
C A
7 15
De los signos siguientes, ¿cual es el más precoz en un Un niño de 6 años, con una talla más corta que la que
síndrome compartimental de los miembros? A) La isquemia debería corresponder a su edad, aqueja dolores moderados
cutánea B) La hipoestesia en el trayecto del nervio afectado de su cadera derecha y claudicación al correr. A la
C) El dolor intenso al estiramiento pasivo muscular D) La exploración se aprecia una discreta atrofia del muslo y
ausencia de pulso E) La debilidad muscular existe una limitación de los movimientos de rotación interna
C y abducción. El diagnostico de sospecha es: A) Sinovitis
8 transitoria de la cadera B) Artritis séptica C) Displasia de
Paciente de 40 años con fractura del cuello del astrágalo, cadera D) Enfermedad de Perthes E) Epifisiolisis de la
con desplazamiento posterior del cuerpo astragalino. cabeza femoral
Señale, de las complicaciones posibles que se relacionan, D
16 administrar analgésicos y solicitar exámenes de sangre y
Un paciente de 30 años sufre una caída a nivel, orina e instalar prótesis parcial de cadera
golpeándose la cara lateral del hombro derecho y D
evolucionando con dolor. Al examen, presenta dolor a la 23
palpación de la zona de contusión, con movilidades pasiva y Las radiografías mínimas que deben solicitarse a un
activa completas, moderadamente dolorosas. La palpación paciente politraumatizado son: A) Cráneo AP, tórax AP y
clavicular y acromial no produce dolor. Se solicitan pelvis AP B) Columna cervical lateral, tórax AP y pelvis AP C)
radiografías de hombro y de articulaciones Cráneo AP, tórax AP y abdomen AP D) Tórax lateral,
acromioclaviculares que resultan normales. La conducta abdomen AP en posición de pie y caderas AP E) Cráneo AP y
más adecuada es: A) Solicitar ecografía de partes blandas y lateral, tórax AP y lateral y pelvis AP
decidir conducta según hallazgos B) Indicar uso de B
cabestrillo y analgésicos orales, mientras persista el dolor 24
C) Indicar yeso braquial y analgesia oral D) Indicar Un paciente de 57 años sufre accidente laboral, sufriendo
analgésicos orales y reposo, sin necesidad de hiperextensión de la rodilla derecha, evolucionando con
inmovilizaciones E) Derivar para resolución quirúrgica dolor y aumento de volumen de dicha rodilla. Al examen se
D aprecia derrame articular, movilidad completa pero
17 dolorosoa. Los bostezos y el cajón posterior son negativos.
¿Cual de las siguientes NO corresponde a una fractura El signo de cajón anterior es positivo. El diagnóstico más
osteoporótica? A) Aplastamientos vertebrales probable es: A) Rotura del menisco medial B) Rotura del
toracolumbares B) Fractura del extremo proximal del fémur menisco lateral C) Rotura del ligamento cruzado anterior D)
C) Fractura del extremo distal del radio D) Fracturas costales Rotura del ligamento cruzado posterior E) Rotura del
E) Fractura subcapital del húmero ligamento colateral medial
D C
18 25
El factor que más incide en la severidad de una fractura Un paciente de 34 años sufre fractura cerrada de pierna, por
expuesta es: A) El sitio de fractura B) El daño a las partes lo que se inmoviliza provisoriamente con una bota corta de
blandas C) El grado de desplazamiento D) El grado de yeso, en espera de cirugía. Evoluciona con intenso dolor. Al
conminución E) El mecanismo de fractura examen físico destaca dolor intenso a la movilización pasiva
B de los ortejos. La conducta inicial más adecuada es; A)
19 Solicitar nueva radiografía de pierna B) Solicitar nuevas
Paciente con dolor lumbar irradiado desde la cara lateral del radiografias de tobillo y pie C) Administrar analgesia
muslo y la pierna hasta el dorso del pie. Al examen físico endovenosa y observar evolución D) Retirar la bota de yeso
presenta reflejos osteotendinosos normales, signo de y elevar la pierna E) Resolver la fractura quirúrgicamente de
Lasegue positivo y claudicación al caminar sobre los inmediato
talones. ¿Cuál es la raíz nerviosa comprometida? A) L3 B) L4 D - Retirar la bota de yeso y elevar la pierna Dx: sindrome
C) L5 D) S1 E) S2 compartimental
C - L5 L3 es cara interna del muslo (con algo de L2, que 26
tiene cara anterior, y L4) S1, S2 son gluteo y cara posterior El mejor examen para la pesquisa precoz de la displasia de
de la pierna, talon y planta del pie cadera en un niño de un mes de edad es: A) Radiografía B)
20 Ecografía C) Tomografía axial computarizada D) Resonancia
Un niño de 5 años, asintomático, es traído por su madre ya magnétia nuclear E) El examen físico
que tiene pie plano. A la exploración no se observa el arco B - Ecografía
plantar, sin embargo este aparece al hiperextender el 27
primer ortejo. Camina sin problemas. La conducta más Una paciente de 67 años tropieza en una escalera, sufriendo
correcta es: A) Tranquilizar a la madre y explicarle que la una inversión forzada del tobillo izquierdo. Evoluciona con
condición de su hijo no le traerá consecuencia alguna B) aumento del volumen, dolor que le impide la marcha y
Explicarle a la madre que su cuadro es causado por un equimosis importante de la zona inframaleolar externa y el
calzado inadecuado y que debe cambiarlo C) Solicitar dorso del pie izquierdo. Al examen presenta dolor a la
radiografías de ambos pies y decidir conducta según palpación de la zona indicada y a la movilización del tobillo.
hallazgos D) Indicar uso de plantillas y controlar cada 3 Los pulsos, la movilidad y la sensibilidad distales son
meses E) Derivar para corrección quirúrgica normales. El examen más adecuado para evaluar la
A - Tranquilizar a la madre y explicarle que la condición de gravedad de la lesión es: A) Radiografías de tobillo B)
su hijo no le traerá consecuencia alguna Ecografía de partes blandas C) Tomografía axial computada
21 D) Resonancia magnética nuclear E) Cintigrafía
Un paciente de 48 años sufre atropello, resultando con A
fractura de la tibia y fémur derechos. Ingresa a pabellón en 28
buenas condiciones, con hemodinamia estable. Ingresa Se Una paciente de 27 años sufre caída sobre hombro derecho,
realiza reducción y osteosíntesis con clavo endomedular, resultando en luxación anterior de dicha articulación, la que
con buen resultado. En el postoperatorio evoluciona con se reduce bajo anestesia profunda. Evoluciona con
compromiso de conciencia y dificultad respiratoria. Al imposibilidad para abducir el hombro y en el examen físico
examen físico se aprecia en Glasgow 10, con frecuencia destaca hipoestesia de la zona deltoidea. ¿Qué estructura
cardiaca de 130x, presión arterial de 100/50mmHg, resultó dañada con mayor probabilidad? A) Nervio mediano
frecuencia respiratoria de 33rpm. El examen pulmonar B) Nervio radial C) Nervio cubital D) Nervio axilar E) Nervio
demuestra crepitos difusos y la saturación arterial por musculocutáneo
pulsometría es 83%. Se observan petequias en el tórax. El D
diagnóstico más probable es: A) Tromboembolismo 29
pulmonar B) Rabdomiolisis aguda C) Endocarditis bacteriana Un paciente de 34 años, consulta por dolor en el cuello,
aguda D) Embolía grasa E) Shock anafiláctico dorso y extremidad superior derecha, que apareció luego de
D La clave la dan los síntomas respiratorios de edema levantar y mover algunos muebles. Al examen presenta
pulmonar, combinados con el rash petequial disminución del reflejo bicipital e hipoestesia en el pulgar
22 derecho. ¿Qué raíz nerviosa es la afectada? A) C5 B) C6 C)
La conducta más adecuada ante una paciente de 80 años, C7 D) C8 E) T1
quien sufre una fractura de la cadera derecha, de tipo B
intertrocantérica y desplazada, es: A) Inmovilizar con yeso 30
pelvipedio y administrar analegesia endovenosa B) Son factores de riesgo para luxación congénita de cadera,
Estabilizar, administrar analgésicos, solicitar exámenes de EXCEPTO: A) Sexo femenino B) Prematurez C) Síndrome de
sangre y orina y realizar inmovilización con yeso pelvipedio Down D) Presentación podálica E) Antecedentes familiares
C) Iniciar analgesia y antibióticos endovenosos y luego de displasia de cadera
realizar cirugía de resección de la cabeza femorl (Cirugía de B
Girdldestone) D) Estabilizar, administrar analgésicos, 31
solicitar exámenes de sangre y orina y realizar osteosíntesis Un paciente de 78 años, con antecedente de hipertensión,
con tornillo dinámico de cadera (DHS) E) Estabilizar, diabetes y demencia inicial por enfermedad de Alzheimer,
sufre una caída a nivel, resultando con imposibilidad para El principal riesgo asociado a las fracturas expuestas en
caminar e intenso dolor. Al examen se parecia extremidad relación a las cerradas, es: A) No unión B) Trombosis venosa
inferior derecha en posición impudica con imposibilidad de C) Infección D) Retraso en la consolidación E)
flectar la cadera. Se solicitan radiografías de caderas que C
demuestran una fractura de la región intertrocantperica, 39
con importante desplazamiento (Tronzo IV). Una vez ¿Cuál de los siguientes es un signo de alarma para un
estabilizado el paciente, la conducta más adecuada es: A) paciente con lumbago? A) Inicio súbito B) Dolor intenso C)
Inmovilizar con yeso pelvipedio B) Realizar osteosíntesis con Irradiación a extremidad inferior D) Contractura muscular E)
placa y tornillos C) Realizar osteosíntsis con DHS (dinamic Antecedente de trauma
hip screw) D) Instalar prótesis parcial de cadera E) Instalar E
prótesis total de cadera 40
C Un paciente de 79 años sufre caída a nivel, quedando con
32 imposibilidad para caminar y mucho dolor. Al examen
Un paciente de 56 años sufre una inversión forzada del pie presenta acortamiento, leve abducción y rotación externa
izquierdo, evolucionndo con importante dolor. Al examen se del muslo derecho, con pulsos y sensibilidad distales
observa edema y equimosis en relación a la zona conservados. Se solicita radiografía de caderas que
inframaleolar derecha, con dolor importante a la palpación demuestra fractura de cuello femoral derecho, desplazada
de dicha zona y a la mayoría de los movimientos del tobillo. (Garden IV). La conducta más adecuada es: A) Inmovilizar
La movilidad, sensibilidad y llenado capilar son normales. La con yeso pelvipedio y administrar analgesia endovenosa B)
conducta más adecuada es: A) Enviar a domicilio indicando Estabilizar, administrar analgésicos, solicitar exámenes de
reposo y analgesia oral B) Solicitar radiografías de tobillo C) sangre y orina y realizar inmovilizaciones con yeso
Solicitar TAC de tobillo D) Inmovilizar con una bota pelvipedio C) Iniciar analgesia y antibióticos endovenosos y
ortopédica E) Inmovilizar con bota corta de yeso, abierta luego realizar cirugía de resección de cabeza femoral
B (Cirugía de Girldestone) D) Estabilizar, administrar
33 analgésicos, solicitar exámenes de sangre y orina y realizar
Un paciente de 25 años cae de un tercer piso, golpeándose osteosíntesis con tornillo dinámico de cadera (DHS) E)
contra el suelo. Al examen se observa muy adolorido, Estabilizar, administrar analgésicos, solicitar exámenes de
consciente, orientado y respira espontáneamente, sin sangre y orina e instalar prótesis parcial de cadera
apremio respiratorio. Al examen presenta frecuencia E
cardíaca de 124lpm, presión arterial 80/40mmHg, 41
frecuencia respiratoria 20rpm. Presenta una fractura en la Un paciente de 34 años presenta dolor cervical, luego de
pierna derecha, con una herida de 4cm, con exposición ósea levantar muebles, que se irradia por la cara lateral del brazo
y sangrado profuso. También se observan varias heridas y el antebrazo, hasta el pulgar. Al examen destaca
abrasivas en la extremidad superior izquierda y en la cara, disminución del reflejo bicipital. La raíz nerviosa
sin sangrado activo. Se instala collar cervical. La conducta comprometida es: A) C5 B) C6 C) C7 D) C8 E) T1
inicial más adecuada es: A) Aplica torniquete en el muslo B
derecho y reponer volumen con suero glucosalino 42
endovenoso B) Realizar compresión sobre la herida de la El agente causal y la vía de llegada más frecuentes, de las
pierna y administrar suero fisiológico por vía venosa osteomielitis agudas, son respectivamente: A) Staph aureus
periférica C) Realizar examen físico y neurológico completo y hematógena B) Staph aureus y por contigüidad C) Staph
D) Realizar intubación orotraqueal y administrar oxígeno E) aureus e inoculación directa D) Strep pyogenes y por
Instalar un catéter venoso central contigüidad E) Strep pyogenes y hematógena
B A
34 43
¿Cuál de las siguientes asociaciones entre patología y Un paciente sufre una caída en bicilceta, golpeándose el
expresión clínica es INCORRECTA? A) Fractura subcapital de hombro derecho. Al examen se aprecia hombro derecho en
húmero - equimosis en la cara interna del brazo B) Fractura charretera, con imposibilidad de abducirlo, asociado a
de escafoides - dolor en la tabaquera anatómica C) Fractura anestesia de la zona deltoidea. El diagnóstico más probable
de cúpula radial - imposibilidad de pronosupinar el es: A) Disyunción acromioclavicular con compromiso del
antebrazo D) Luxación posterior de cadera - posición púdica nervio radial B) Disyunción acromioclavicular con
E) Luxación anterior del hombro - signo de la tecla de piano compromiso del nervio axilar o circunflejo C) Luxación
E anterior de hombro con compromiso del nervio del nervio
35 radial D) Luxación anterior de hombro con compromiso del
La complicación más habitual de la fractura del cuello del nervio del nervio axilar o circunflejo E) Luxación anterior de
astrágalo es: A) Lesión de la arteria tibial posterior B) hombro con compromiso del nervio musculocutáneo
Necrosis avascular C) Síndrome compartimental del pie D) D
Consolidación viciosa E) Osteomielitis 44
B ¿Cuál de las siguientes asociaciones es INCORRECTA? A)
36 Fractura de escafoides - dolor en la tabaquera anatómica B)
Un hombre de 29 años sufre una lesión traumática del Fractura de cúpula radial - imposibilidad de pronosupinar la
hombro izquierdo, presentando una luxación anterior de muñeca C) Fractura de Colles - radialización, impactación y
hombro, la que es tratada ortopédicamente con buena desviación dorsal D) Fractura de húmero proximal -
evolución. En una segunda instancia presenta otra luxación equimosis en la cara medial del brazo E) Fractura de la
anterior del hombro izquierdo, en relación a un traumatismo tuberosidad del olecranon - imposibilidad de flectar el codo
de poca energía, la que es reducida adecuadamente. La E
conducta más adecuada es: A) Inmovilizar con cabestrillo 45
simple B) Inmovilizar con cabestrillo bloqueado C) La no unión hipertrófica se debe a: A) Isquemia B)
Inmovilizar con vendaje en ocho D) Inmovilizar con yeso Inadecuada analgesia C) Inmovilización insuficiente D)
escápulobraquial E) Realizar reparación quirúrgica Interposición de partes blandas E) Mal alineamiento
E C
37 46
Un paciente sufre una fractura de húmero, con lesión del Los ejercicios que se indican como parte del tratamiento de
nervio radial. ¿Cuál de los siguientes signos es más la disfunción patelo-femoral, está dirigidos principalmente
probable de encontrar en el examen físico de este paciente? a: A) Fortalecer el músculo vasto medial B) Fortalecer el
A) Imposibilidad para flexionar la muñeca B) Imposibilidad músculo vasto lateral C) Fortalecer el músculo recto femoral
para extender la muñeca C) Anestesia de los pulpejos del D) Fortalecer los músculos aductores E) Fortalecer el
índice y dedo medio D) Anestesia de los pulpejos del músculo bíceps femoral
meñique y anular E) Anestesia de la palma de la mano A
B 47
38 Las fracturas de tobillo, se producen mayormente por: A)
Caídas de altura B) Golpe directo C) Inversión o rotaciones
forzadas D) Fracturas sobre hueso patológico E) Accidentes una fractura expuesta del antebrazo de 30 minutos de
de tránsito evolución. No presenta sangramiento importantes y sus
C signos vitales son normales. ¿Cuál de las siguientes
48 medidas es MENOS adecuada para el manejo de este
Identifique la asociación correcta: A) Rotura del ligamento paciente? A) Derivación a un centro de mayor complejidad
cruzado posterior - Signo de bostezo B) Esguince de B) Aseo con abundante suero fisiológico C) Antibióticos
ligamento colateral lateral C) Rotura del ligamento cruzado endovenosos de amplio espectro D) Vacunación antitetánica
anterior - signo de cajón posterior D) Lesión del menisco E) AINEs endovenosos
medial - dolor a la compresión axial, con rotación externa de A
la pierna E) Esguince del ligamento colateral lateral - 57
hemartrosis Respecto a la displasia de cadera es FALSO que: A) Es más
D frecuente en mujeres y en niños con antecedente de
49 presentación podálica B) Uno de cada 1.000 niños la
Un recién nacido, de 14 días de vida, presenta signo de presentan C) El signo de Bartow se caracteriza por la
Ortolani bilateral. La conducta más adecuada es: A) Solicitar posibilidad de luxar la cadera del recién nacido, mientras
radiografía de pelvis AP inmediatamente B) Solicitar que el de Ortolani reposiciona la cadera que está luxada D)
ecografía de caderas, inmediatamente C) Solicitar La radiografía es el examen de elección para el diagnóstico
radiografía de pelvis AP, al cumplir tres meses de vida D) del recién nacido E) Los niños que se diagnostican después
Solicitar radiografía de pevis AP, al cumplir 6 meses de vida de los 18 meses de edad suelen ser tratados de manera
E) Solicitar ecografía de caderas, al cumplir tres meses de quirúrgica
vida D
B 58
50 Un paciente de 40 años presenta aumento de volumen en
Un hombre de 30 años consulta por dolor en la rodilla relación a la rodilla derecha que ha crecido lentamente a lo
izquierda, de 5 días de evolución, que aparece al caminar y largo del último año y que presenta dolor en algunas
que se inició durante una expedición, en la que caminó ocasiones. Al examen físico se aprecia aumento de volumen
largas distancias. Al examen físico presenta dolor a la de consistencia ósea. Se realiza una radiografía que
palpación de la cara interna de la zona tibial proximal de la muestra un tumor óseo de 12cm de diámetro,
pierna izquierda. El diagnóstico más probable es: A) multiloculado, de aspecto irregular, que compromete la
Tendinitis rotuliana B) Tendinitis ansariana C) Disfunción arquitectura de la epífisis distal del fémur. El diagnóstico
patelofemoral D) Bursitis E) Síndrome de Osgood Schlater más probable es: A) Osteocndroma B) Osteoma osteoide C)
(entesitis rotuliana) Tumor de células gigantes D) Tumor de Ewing E)
B Osteosarcoma
51 C
Un paciente de 45 años sufre una fractura de femur, por 59
arma de fuego, la que es manejada quirúrgicamente con un ¿Qué fractura supone una mayor gravedad? A) Fractura
tutor externo y con antibioticoterapia. Presenta importante expuesta de fémur, con herida de 5cm, debida a caída de
dolor, el que es manejado con AINEs, con escasa respuesta. altura B) Fractura expuesta de fémur, con pequeña herida,
Algunos días después, presenta dolor intenso de toda la debido a impacto de bala C) Fractura expuesta de tobillo, de
extremidad y al examen físico se constata dolor a la 2 horas de evolución, por golpe directo D) Fractura expuesta
movilización de los dedos, con disminución de los vellos y de pierna, con herida de gran tamaño y mínima cobertura
adelgazamiento de la piel del pie. Presenta pulsos, de partes blandas, debida a accidente de tránsito E)
movilidad y sensibilidad distal conservados. El diagnóstico Fractura expuesta de antebrazo, con restos de tierra en la
más probable es: A) Síndrome compartimental B) Trombosis herida, por caída en bicicleta
venosa profunda C) Lesión de la arteria femoral D) Distrofia D
smpática refleja E) Retracción isquémica de Volkman 60
D Una niña de 12 años, asintomática, presenta un test de
52 Adams que muestra cierto grado de asimetría en el tronco y
Un paciente de 45 años sufre una caída de altura, cayendo se visualiza una giba costal leve. Se solicitan radiografías de
sobre la extremidad superior derecha y resultando con columna que muestran una escoliosis con desviación de
fractura del tercio medio de la diáfisis humeral. ¿Qué 10%. La conducta más adecuada es: A) Indicar ejercicios
estructura se afectará con mayor probabilidad? A) Nervio posturales y observar evolución B) Solicitar TAC de columna
axilar B) Nervio mediano C) Nervio radial D) Nervio cubital C) Indicar uso de corsé dorsolumbar D) Indicar vendaje en
E) Nervio musculocutáneo ocho E) Resolución quirúrgica
C A
53 61
¿Por qué mecanismo se produce la mayor parte de las Un paciente presenta dolor lumbar de inicio súbito, que
fracturas de cadera? A) Caídas a nivel B) Caídas de altura C) inició al movilizar unos muebles y que e irradia por la cara
Rotación forzada D) Fractura espontánea durante la narcha anterior del muslo y la cara medial de la pierna, hasta el
E) Accidentes de tránsito maléolo medial. Al examen físico destaca disminución del
A reflejo rotuliano. La raíz comprometida es: A) L1 B) S1 C) L2
54 D) L4 E) L5
¿Cuál de las siguientes asociaciones es incorrecta? A) D
Distrofia simpático refleja - disminución de los pulsos 62
distales B) Embolia grasa - rash petequial C) Embolia grasa - Respecto a la epitrocleítis es verdadero que: A) Se le llama
edema pulmonar no cardiogénico D) Distrofia simpático codo del tenista B) Se caracteriza por dolor a la palpación
refleja - cambios en la piel y vellos distales E) Síndrome de la inserción proximal de los músculos flexores del
compartimental - dolor a la movilización distal antebrazo C) Su tratamiento suele ser quirúrgico D)
A Frecuentemente se asocia a síndrome de atrapamiento del
55 nervio cubital E) Es más frecuente que la epicondilitis
¿Cuál de los siguientes es un signo de mayor gravedad ante B
un traumatismo craneal? A) Herida de cuero cabelludo B) 63
Salída de líquido transparente por un óido C) Hematoma de ¿Cuál es la presentación clínica más habitual de las
cuero cabelludo en la frente D) Convulsión de 30 segundos fracturas de cadera? A) Acortamiento, rotación interna y
inmediatamente después del golpe E) Desorientación de 4 aducción del muslo, con imposibilidad de caminar B)
minutos de duración, después del golpe, seguido de Acortamiento, rotación externa y abducción del muslo, con
amnesia de esos momentos imposibilidad de caminar C) Acortamiento, rotación interna
B y abducción del muslo, con imposibilidad de caminar D)
56 Posición púdica, equimosis, crujido articular y dolor intenso
Un paciente de 35 años consulta en el consultorio de durante la marcha E) Posición impúdica, aumento de
Futalefú (zona rural), donde usted es el médico de turno. volumen y dolor intenso durante la marcha
Sufrió una caída a caballo hace 30 minutos, resultando con B
64 C
Un paciente sufre un golpe directo sobre la pierna derecha, 73
al ser barrido mientras jugaba fútbol. Evoluciona con dolor ¿Qupe fractura puede ser considerada osteoporótica? A)
intenso de la rodilla derecha, que le impide caminar. Al Fractura de cráneo B) Fractura subcapital de húmero C)
examen físico se aprecia derrame articular y destaca signo Fractura de falange D) Fractura de diáfisis femoral E)
de cajón posterior. Las radiografías no muestran lesiones Fractura de escápula
óseas. El diagnóstico más probable es: A) Lesión del B
menisco medial B) Rotura del ligamento cruzado anterior C) 74
Rotura del ligamento cruzado posterior D) Rotura del Un paciente refiere dolor lumbar irradiado a la pierna
ligamento colateral medial E) Rotura del ligamento colateral izquierda. El dolor recorre la cara anterior y medial del
lateral muslo y al examen físico se constata debilidad a la
C extensión de la rodilla y disminución de la intensidad del
65 reflejo rotuliano. La raíz nerviosa afectada es: A) L1 B) L3 C)
¿Cuál de las siguientes fracturas suele producirse L5 D) S1 E) S3
secundaria a osteoporosis? A) Fractura de cúpula radial B) B En otras preguntas han colocado L4... which is it? Aunque
Fractura de diáfisis femoral C) Fracturas costales D) Fractura la pregunta anterior llevaba hasta maleolo medial...
de extremo distal del radio E) Fractura de tobillo 75
D Un paciente de 40 aos, sin antecedentes de importancia,
66 consulta por dolor muy intenso en la zona sdorsal alta, de 4
Un paciente sufre hiperextensión forzada del dedo índice horas de evolución, que inició de manera súbita luego de
derecho, evolucionando con dolor, aumento de volumen y levantar una caja. Al examen físico presenta signos vitales
equimosis, que limita los movimientos del dedo, normales y se constata dolor importante a la flexión y
especialmente la flexión de la articulación interfalángica rotación lateral a derecha de columna dorsal, sin otras
proximal. Se solicita una radiografía anteroposterior y alteraciones. La conducta más adecuada es: A) Rx de
lateral que no detecta lesiones óseas. El diagnostico más columna lumbar B) Ecografía de partes blandas C) TAC de
probable es: A) Esguince de dedo B) Fractura avulsiva de la columna dorsal D) Solicitar RMN de columna E) Indicar
base de la falange media C) Fractura avulsiva del extremo AINEs y reposo relativo
distal de la falange proximal D) Luxación interfalángica E
proximal E) Dedo en gatillo 76
A El tratamiento del síndrome compartimental secundario a
67 una fractura, suele ser: A) Analgesia B) Anticoagulación C)
Un paciente sufre una fractura de la diáfisis humeral, por un Vendaje compresivo D) Inmovilización adecuada E)
golpe directo. ¿Qué estructura estará lesionada con mayor Fasciotomía amplia
posibilidad? A) Nervio cubital B) Arteria braquial C) Nervio E
mediano D) Vena basílica E) Nervio radial 77
E ¿Qué fractura es característica de la edad pediátrica? A)
68 Fracturas en tallo verde B) Fractura conminuta C) Fractura
Una paciente de 27 años cae de una bicicleta, golpeándose de escafoides D) Fractura de muñeca E) Fractura de Garden
el hombro derecho. Evoluciona con dolor, aunque mantiene A
gran parte de la movilidad. Al examen físico se observa 78
equimosis de la zona del golpe y se observa el signo de la Un joven de 17 años cae del tercer piso de su colegio,
tecla de piano. La imagen más adecuada para evaluar a mientras se encontraba en estado de ebriedad. Usted llega
esta paciente es: A) TAC de hombro derecho B) Ecografía de en una ambulancia y constata que respira y realiza algunos
hombro derecho C) Rx lateral de hombro derecho D) Rx AP movimientos, sin responder cuando se le habla. Además, se
bilateral comparativa de hombros E) Radiografía axial de aprecia una fractura expuesta de la pierna izquierda, que
hombro derecho sangra activamente y una herida en el cuero cabelludo. La
D Signo de tecla del piano corresponde a luxación de primera medida a realizar A) Instalar una vía venosa B)
clavícula Realizar compresión de las heridas para detener la
69 hemorragia C) Inmovilizar la columna cervical D) Realizar
Una paciente de 68 años, alcohólica, consulta por dolor de intubación orotraqueal E) Retirar la ropa para poder evaluar
la ingle izquierda, que inició el día de ayer y que le dificulta la totalidad de las lesiones
la marcha. Al examen físico se aprecia en buenas C
condiciones generales, con signos vitales normales y 79
presenta importante dolor a la movilización del muslo Las fracturas con rasgo intraarticular habitualmente se
izquierdo, especialmente ante la abducción con rotación manejan con cirugía, debido a que en caso contrario
externa. El diagnóstico más probable es: A) Artritis séptica desarrollan: A) Necrosis avascular B) Artrosis C)
B) Artrosis C) Síndrome de pellizcamiento acetabular D) Pseudoartrosis o no unión D) Hemartrosis y dolor
Pelviespondiloartropatía E) Necrosis avascular de la cabeza persistente E) Artritis séptica
femoral B
E 80
70 Un paciente de 34 años sufre una caída mientras jugaba
¿Qué sería MENOS probable de hallar en una fractura de fútbol, en la que sufrió una torsión de la rodilla izquierda. Al
cadera clásica? A) Imposibilidad de levantar el talón del examen físico presenta signos de derrame articular y es
suelo B) Abducción del muslo C) Acortamiento del muslo D) posible luxar la rodilla movilizando la tibia hacia anterior. El
Rotación externa del muslo E) Equimosis en la cara externa diagnóstico más probable es: A) Rotura del tendón rotuliano
del muslo B) Rotura del ligamento cruzado anterior C) Rotura del
E ligamento cruzado posterior D) Rotura del ligamento
71 colateral medial E) Rotura del ligamento colateral lateral
Un paciente sufre un accidente de tránsito con luxofractura B
de la rodilla izquierda. Evoluciona a los 5 minutos con dolor 81
importante en la pierna y pie izquierdos. Al examen físico se Un paciente de 72 años presenta una fractura de cadera de
constata palidez, llene capilar muy enlentecido y ausencia cuello femoral desplazada, tipo Garden IV. La conducta más
de pulso pedio y tibial posterior. El diagnóstico más adecuada es: A) Estabilizar al paciente, administrar
probable es: A) Síndrome compartimental B) Lesión de la analgésicos y derivar para resolución quirúrgica con prótesis
arteria poplítea C) Trombosis venosa profunda D) Distrofia parcial de cadera B) Estabilizar al paciente, administrar
simpático refleja E) Rotura de quiste de Baker analgésicos y derivar para resolución quirúrgica con tornillo
B canulado C) Estabilizar al paciente, administrar analgésicos
72 y derivar para resolución quirúrgica con tutor externo D)
¿Qué fracturas son frecuentes y deben buscarse Estabilizar al paciente, administrar analgésicos y derivar
dirigidamente en las caídas de pie desde altura? A) Cadera para inmovilización con bota larga de yeso E) Estabilizar al
y fémur B) Tibia y fémur C) Columna y calcáneo D) Calcáneo paciente, administrar analgésicos y derivar para
y femur E) Tibia y calcáneo inmovilización con yeso pelvipedio
A un clavo endomedular. Al día siguiente evoluciona con dolor
82 y edema de la extremidad. Al examen físico presenta dolor
Un paciente de 56 años consulta por dolor en el talón marcado a la palpación muscular de la pantorrilla, con
izquierdo, que aparece al caminar y que le dificulta la normalidad de los pulsos, sensibilidad y movilidad distal. El
marcha, cuando es mayor a 3 cuadras. Se solicita una diagnóstico más probable es: A) Sindrome compartimental
radiografía de retropie que visualiza un espolón calcáneo de B) Lesión de arteria poplitea C) Distrofia simpático-refleja D)
1cm. La conducta más adecuada es: A) Indicar uso de Trombosis venosa profunda E) Evolución postquirúrgica
calzado cómodo o taloneras B) Iniciar corticoides orales C) normal
Realizar resección quirúrgica D) Solicitar TAC del retropié E) D
Solicitar ecografía del retropié 91
A Un paciente sufre una caída mientras andaba en bicicleta,
83 golpeándose el hombro izquierdo. Al examen presenta
Identifique la asociación incorrecta entre el tumor óseo y su "hombro en charretera" e impotencia funcional. El
características: A) Osteocondroma - reacción perióstica en diagnóstico más probable es: A) Fractura de clavícula B)
sol naciente B) Osteosarcoma - discontinuación de la Disyunción acromioclavicular C) Luxación anterior del
cortical ósea C) Osteoma osteoide - dolor mayor en la noche hombro D) Luxación posterior del hombro E) Fractura
D) Sarcoma de células gigantes - rápido crecimiento y subcapital del hombro
destrucción ósea local E) Sarcoma de Ewing - rápido C
crecimiento en ubicación diafisaria 92
A Un hombre presenta dolor lumbar irradiado a la extemidad
84 inferior izquierda, por la cara lateral de la rodilla y anterior
Un paciente presenta una inversión forzada del pie de la pierna, hasta los ortejos. Al examen tiene debilidad
izquierdo mientras bajaba una escalera, evolucionando con para abducir el muslo y extender el primer ortejo. Los
intenso dolor. Al examen físico se aprecia equimosis y reflejos son normales. ¿Que raíz está afectada? A) L3 B) L4
aumento de volumen en la zona inframaleolar externa. Se C) L5 D) S1 E) S2
solicitan radiografías de tobillo y piernas, que no visualizan C
lesiones óseas. La conducta más adecuada es: A) Indica 93
analgesia e inmovilizar con bota larga de yeso por al menos Los agentes etiológicos más frecuentes de la osteomielitis
3 meses y controlar con radiografías periódicas cada 2 aguda y de las artritis sépticas son, respectivamente: A)
semanas B) Indicar analgesia, reposo relativo y el uso de Staph aureus y staph aureus B) Staph aureus y gonococo C)
una bota ortopédica removible C) Indicar analgesia y Strep beta hemolitico y gonococo D) Strep beta hemolitico y
prohibir el apoyo de la extremidad por al menos 30 días D) staph aureus E) Strep beta hemolitico y strep beta
Indicar analgesia y ejercicios de rotación e inversión del hemolítico
tobillo y controlar en 7 días E) Solicitar TAC de tobillo A
B 94
85 Un paciente de 50 años consulta por dolor en la pierna
Un paciente de 23 años presenta una caída a nivel, cayendo izquierda. Al examen se aprecia aumento de volumen en
sobre la extremidad superior izquierda, en extensión. Al relación a la tibia. Se solicita radiografía de pierna que
examen físico presenta dolor a la presión de la tabaquera visualiza un tumor óseo de 5cm de diámetro, con
anatómica. El diagnóstico más probable es: A) Fractura de compromiso de la arquitectura, reacción perióstica en tela
trapecio B) Fractura de la cúpula radial C) Fractura de la de cebolla y disrupción de la cortical. El diagnóstico más
apófisis estiloides D) Fractura de escafoides E) Fractura de probable es: A) Osteosarcoma B) Osteoma osteoide C)
extremo distal del radio Osteocondroma D) Adantimoma E) Enfermedad de Paget
D A
86 95
Un hombre sufre un accidente laboral; resultando con Un adolescente de 14 años con sobrepeso, consulta por
múltiples fracturas en la extremidad superior izquierda. Al dolor inguinal izquierdo intenso, que le impide la marcha. Al
examen presenta imposibilidad de extender el codo y la examen físico presenta dolor y limitación a la movilización
muñeca. ¿Qué estructura está presuntamente dañada? A) de la cadera izquierda, especialmente a la abducción y
Nervio mediano B) Nervio cubital C) Nervio musculocutáneo rotación interna. El diagnóstico más probable es: A)
D) Nervio radial E) Nervio axilar Epifisiolisis de la cabeza femoral B) Artrosis C) Sinovitis
D transitoria D) Enfermedad de Perthes E) Luxación congénita
87 de cadera (displasia de cadera)
Un paciente sufre una caída mientras jugaba fútbol, con un A
golpe en la rodilla izquierda. Al examen físico se aprecian 96
signos de derrame articular, el que se punciona dando Un paciente de 67 años consulta por dolor en la ingle
salida a sangre con gotas de grasa. El diagnóstico más izquierda, que aparece al caminar y que en ocasiones se
probable es: A) Rotura meniscal B) Rotura de ligamento presenta en el reposo, exacerbándose en los días en que
cruzado C) Rotura del ligamento colateral D) Rotura del realiza actividad física. Al examen físico presenta movilidad
cartílago articular E) Fractura de rodilla completa de la rodilla y cadera izquierdas, pero presenta
E dolor a la abducción completa de la cadera. Se solicita una
88 radiografía que muestra disminución del espacio articular,
Un paciente de 80 años sufre una caída a nivel, resultando con presencia de osteofitos acetabulares. La conducta más
con imposibilidad de ponerse de pie y dolor. Al examen adecuada es: A) Resolver quirúrgicamente B) Iniciar
físico se aprecia posición impúdica de la extremidad inferior paracetamol C) Realizar infiltración articular con corticoides
izquierda. El diagnóstico más probable es: A) Luxación D) Solicitar TAC de cadera E) Iniciar ibuprofeno
anterior de la cadera B) Luxación posterior de la cadera C) B
Fractura de cadera D) Fractura de acetábulo E) Lesión del 97
nervio ciático ¿Qué antibiótico indicaría en un paciente con una fractura
C expuesta traumática, con mala cobertura de partes blandas
89 y presencia de restos vegetales en la herida? A) Cloxacilina
Un paciente de 19 años cae a nivel, con apoyo sobre la B) Imipenem C) Clindamicina + cloxacilina D) Cefazolina +
mano derecha, evolucionando con intenso dolor e gentamicina + metronidazol E) Ceftriaxona + gentamicina
impotencia funcional en el codo, especialmente con los + metronidazol
movmientos de pronosupinación. El diagnóstico de D
sospecha es: A) Fractura de cúpula radial B) Fractura de 98
olécranon C) Luxación de codo D) Lesión del nervio cubital Un adulto mayor de 80 años consulta por dolor lumbar de
E) Fractura de antebrazo larga data, irradiado a ambas piernas que se presenta al
A caminar y que lo obliga a detenerse luego de andar una
90 cuadra. Refiere que cede al sentarse y que es menos
Un paciente de 45 años presenta una fractura de pierna, la intenso cuando camina inclinado hacia delante. El
que es reducida y fijada quirúrgicamente, con instalación de diagnóstico más probable es: A) Espondilosis lumbar B)
Enpondilolistesis lumbar C) Raquiestenosis D) Lumbago Epifisiolisis de la cabeza femoral B) Enfermedad de Osgood
tumoral E) Espondilitis anquilosante Schlatter C) Enfermedad de Perthes D) Sarcoma de Ewing E)
99 Osteomielitis
Un paciente de 45 años sufre un accidente de tránsito, de A
alta energía. Al examen físico está consciente, respira 106
normalmente y se aprecia una fractura expuesta en la Una mujer de 80 años sufre una caída a nivel, golpeándose
pierna izquierda, que sangra activamente. Se instala collar contra el suelo y presentando mucho dolor e imposibilidad
cervical y vía venosa periférica con administración de suero de caminar. Al examen se aprecia rotación externa y
fisiológico. ¿Qué conducta es la más adecuada para el abducción del muslo izquierdo. El diagnóstico más probable
manejo de la hemorragia? A) Realizar compresión con es: A) Fractura de pelvis B) Esguince de cadera C) Fractura
apósitos B) Realizar un torniquete con una banda elástica de cadera D) Necrosis avascular de la cabeza femoral E)
removible C) Realizar aseo con suero fisiológico D) Realizar Luxación de cadera
ligadura arterial E) Realizar reducción de la fractura y C
estabilización con una férula neumática 107
A Un paciente sufre torsión de rodilla izquierda mientras
100 jugaba basquetbol, presentando dolor importante. Al
Un paciente sufre una torsión de rodilla mientras jugaba examen físico presenta dolor al valgo forzado y a la
basquetbol, evolucionando con dolor en la cara lateral de la palpación de la cara medial de la rodilla izquierda. La
rodilla, sin impedimento para la marcha. Al examen físico se estructura más probablemente lesionada es: A) Ligamento
aprecia dolor a la palpación de la cara lateral de la rodilla y cruzado anterior B) Ligamento cruzado posterior C)
al valgo forzado, sin signos de inestabilidad ni de derrame Ligamento colateral medial D) Menisco medial E) Tendón de
articular. La maniobra de Appley es negativa. El diagnóstico la pata de ganso
más probable es: A) Lesión del menisco medial B) Lesión del C
ligamento cruzado anterior C) Lesión del ligamento cruzado 108
posterior D) Lesión del ligamento colateral lateral E) Lesión El tratamiento antibiótico de la osteomielitis aguda suele
del tendón de la pata de ganso ser: A) Cloxacilina endovenosa B) Gentamicina endovenosa
D C) Vancomicina endovenosa D) Ceftriaxona endovenosa E)
101 Cefazolina + metronidazol endovenosos
El concepto de "pseudoartrosis" se refiere a: A) Dolor A
articular crónico, no inflamatorio, sin cambios en la 109
radiología B) La movilidad anormal de un segmento óseo Un paciente de 40 años sufre un accidente laboral, en un
por interposición de partes blandas durante el proceso de fundo agrícola, presentando una fractura expuesta de la
consolidación de una fractura C) La pérdida de pierna izquierda, con una herida de 8cm y visualización de
funcionalidad de una articulación con imposibilidad de hueso. Usted la recibve en la posta rural dodne es el único
realizar ninguno de los movimientos habituales de dicha médico y que cuenta solo con un pabellón básico de cirugía
articulación D) La consolidación de una fractura en la cual menor. Al examen fisico se observan restos de tierra y
hay un mal alineamiento de los ejes, ya sea por vegetales en la herida. La conducta más adecuada es: A)
desplazamientos anteroposteriores, laterales o rotación del Derivar de inmediato a un centro de mayor complejidad B)
segmento óseo distal E) El compromiso articular Realizar aseo con suero fisiológico, aplicar vacunación
inflamatorio, con compromiso asimétrico del cartílago antitetánica y derivar posteriormente a unc centro de mayor
articular y presencia de lesiones óseas en "sacabocado" complejidad C) Realizar aseo con suero fisiológico, aplicar
B No es falsa artrosis, es condición patológica de falsa vacunación antitetánica, administrar cloxacilina endovenosa
articulación y analgésicos, inmovilizar de manera transitoria y derivar
102 posteriormente a un centro de mayor complejidad D)
Un niño de 30 días de vida presenta signo de Ortolani Realizar aseo con suero fisiológico, aplicar vacunación
positivo. La conducta más adecuada es: A) Solicitar antitetánica, administrar cloxacilina + gentamicina +
radiografías de cadera de inmediato B) Solicita ecografía de metronidazol por vía endovenosa y analgésicos, inmovilizar
cadera de inmediato C) Observar evolución, controlando de manera transitoria y derivar posteriormente a un centro
periódicamente D) Soliciar radiografías de cadera al cumplir de mayor complejidad E) Realizar aseo con suero fisiológico,
los 3 meses de edad E) Iniciar tratamiento con correas de aplicar vacunación antitetánica, administrar ceftriaxona +
Pavlik metronidazol por vía endovenosa y analgésicos, suturar la
B herida, inmovilizar con una valva de yeso cerrada y derivar
103 posteriormente a un centro de mayor complejidad
Un hombre de 30 años sufre una fractura de brazo, la que D
es manejada ortopédicamente. Tres meses después ya no 110
siente dolor, sin embargo se aprecia movilidad anormal en Un paciente de 40 años consulta por dolor en el codo
el tercio medio del brazo, logrando desplazar de manera derecho, que aparece con algunos movimientos. Al
independiente la zona distal a la fractura, en relación a la examinarlo se constata dolor a la palpación de la inserción
proximal. El diagnostico es: A) Consolidación viciosa B) de los músculos flexores de la muñeca. El diagnóstico más
Retraso en la consolidación C) Fractura en hueso patológico probable es: A) Bursitis B) Epicondilitis C) Epitrocleitis D)
D) Necrosis avascular E) Pseudoartrosis Tendinitis bicipital E) Tendinitis tricipital
E C
104 111
Un paciente de 18 años sufre una caída mientras jugaba Un paciente de 80 años sufre una caída a nivel, apoyándose
fútbol, golpeándose el hombro izquierdo contra el suelo, sobre la mano derecha, evolucionando con intenso dolor a
evolucionando con dolor y limitación de los movimientos. Al nivel del hombro y la zona proximal del brazo. Al examen
examen físico se aprecia el signo de la tecla de piano. El físico se observa intenso dolor a la movilización activa y
diagnóstico más probable es: A) Luxación anterior del pasiva y un área de equimosis en la cara interna del brazo
hombro B) Luxación posterior del hombro C) Lesión del derecho. El diagnóstico más probable es: A) Luxación
nervio axilar D) Disyunción acromioclavicular E) Fractura de anterior de hombro B) Luxofractura de hombro C) Fractura
hombro desplazada subcapital humeral D) Fractura de diáfisis humeral E)
D Fractura supracondílea del húmero
105 C
Un paciente de 12 años, sexo masculino, sin antecedentes 112
médicos de importancia, practica deporte en forma regular, Un paciente sufre una fractura de cabeza del peroné, por un
fundamentalmente fútbol y tenis. Acude a la consulta de su impacto directo. ¿Qué estructura será lesionada con mayor
pediatra por historia de dolor en la ingle, rodilla y muslo probabilidad? A) Nervio interóseo posterior B) Nervio
izquierdos de dos meses de evolución, con claudicación de peroneo común C) Nervio ciático D) Nervio femoral E)
la misma extremidad en el último mes. Al examen físico Nervio interóseo anterior
destaca un niño obeso, con ligera rotación externa de la B
extremidad inferior izquierda y limitación de la rotación 113
interna por dolor. El diagnóstico más probable: A)
Un paciente sufre un accidente de tránsito, resultando con Un adolescente de 11 años presenta algún grado de
múltiples traumatismos en todo el cuerpo. ¿Qué radiografías desviación del tórax, se realiza un test de Adams que
deben solicitarse como mínimo de manera dirigida? A) demuestra la presencia de una giba costal, corroborándose
Cráneo AP, tórax AP y abdomen AP B) Cráneo lateral, tórax radiológicamente una escoliosis con 20° de desviación. La
AP y pelvis AP C) Cuello lateral, tórax AP y pelvis AP D) conducta más adecuada es: A) Tratar con ejercicios
Cuello lateral, abdomen AP y pelvis AP E) Tórax AP, posturales y de fortalecimiento de la musculatura posterior
abdomen AP y pelvis AP B) Tratar con correas de Pavlik C) Tratar con un corsé
C removible D) Tratar de manera quirúrgica E) Enviar a
114 domicilio y explicar que se corregirá sola, sin necesidad de
Un niño de 8 añs suufre una fractura de antebrazo, la que tomar ninguna medida
se maneja ortopédicamente. Inicialmente presenta intenso A
dolor, por lo que se maneja con analgésicos en altas dosis. 123
Algunas semanas después presenta dificultad para mover La mejor imagen para evaluar un aumento de volumen de
los dedos con la "mano en garra". El diagnóstico más una extremidad, compatible con un sarcoma de partes
probable es: A) Distrofia simpático refleja B) Mala unión C) blandas es: A) Ecografía de partes blandas B) Cintigrafía C)
Trombosis venosa profunda D) Lesión del nervio mediano E) Tomografía axial computariada D) RMN E) Rx simples
Retracción isquémica de Volkman seriadas
E D
115 124
¿Que fracturas son frecuentes y deben buscarse ¿Cuál de las siguientes situaciones requiere ser resuelta en
dirigidamente en un paciente que cae de pie desde 3 primer lugar en un paciente politraumatizado grave? A)
metros de altura? A) Rodilla y cadera B) Cadera y columna Neumotórax a tensión B) Contusión cerebral C) Perforación
C) Calcáneo y tobillo D) Columna y calcáneo E) Rodilla y de víscera hueca abdominal D) Fractura expuesta de fémur
tobillo E) Luxofractura vertebral toracolumbar
D A
116 125
Un paciente sufre una fractura del cuello del astrágalo. La Un hombre de 30 años sufre una fractura de brazo,
complicación asociada a esta lesión es: A) Fractura de la presentando además imposibilidad de extender el codo y la
base del 5to metatarsiano B) Lesión de los extensores de los muñeca. La estructura dañada es: A) Nervio circunflejo B)
ortejos C) Lesión de la arteria pedia D) Necrosis avascular E) Nervio radial C) Nervio cubital D) Nervio musculocutáneo E)
Síndrome compartimental Nervio mediano
D B
117 126
Un paciente de 60 años presenta dolor en la zona proximal Un paciente de 18 años sufre una caída mientras jugaba
del muslo. Se solicita una radiografía que muestra un tumor fútbol, golpeándose el hombro izquierdo contra el suelo,
de 10cm de diámetro, con compromiso de partes blandas, evolucionando con dolor y limitación de los movimientos. Al
disrupción de la cortical y reacción periostica en tela de examen físico se aprecia pérdida de la convexidad del
cebolla. El diagnóstico más probable es: A) Tumor de células hombro, observándose como un "ángulo recto". El
gigantes B) Sarcoma de Ewing C) Osteosarcoma D) diagnóstico más probable es: A) Luxación anterior de
Osteocondroma E) Quiste óseo simple hombro B) Luxación posterior de hombro C) Lesión del
C nervio axilar D) Disyunción acromioclavicular E) Fractura de
118 hombro desplazada
Un niño de 3 meses de edad presenta signo de Barlow y se A
constata limitación a al abducción de la cadera izquierda. La 127
conducta más adecuada es: A) Solicitar radiografía AP de Una mujer de 80 años sufre un accidente de tránsito,
pelvis B) Solicitar radiografía de la cadera izquierda C) golpeándose la rodilla y presentando mucho dolor e
Solicitar ecografía de caderas D) Solicitar RMN de caderas imposibilidad de caminar. Al examen presenta movilidad
E) Resolver quirurgicamente de inmediato normal de la rodilla izquierda, pero se aprecia rotación
A La ecografía es util en el recién nacido, luego del 2-5to interna y aducción del muslo izquierdo y dolor importante a
mes se usa Rx los movimientos de la cadera izquierda. El diagnóstico más
119 probable es: A) Fractura de pelvis B) Fractura de diáfisis
¿Cuál es la conducta ante un paciente con artrosis de femoral C) Fractura de cadera D) Necrosis avascular de la
cadera izquierda, que no responde al tratamiento con cabeza femoral E) Luxación posterior de cadera
ejercicios y paracetamol 1 gramo cada 8 horas? A) E
Aumentar a dosis a 2g cada 8 horas B) Agregar ibuprofeno 128
C) Agregar prednisona D) Realizar infiltraciones con Un paciente sufre "torsión" de la rodilla izquierda, al ser
corticoides E) Resolver quirúrgicamente golpeado por otro jugador, mientras jugaba basquetbol,
B presentando dolor importante e inestabilidad de la rodilla. Al
120 examen físico presenta dolor, singos de derrame articular y
Un paciente sufre una "torsión del pie izquierdo" mientras la posibilidad de luxar la rodilla, desplazando l tibia hacia
caminaba, con inversión forzada, evolucionando con dolor anterior. La estructura más probablemente lesionada es: A)
que le impide caminar. Al examen físico se aprecia Ligamento cruzado anterior B) Ligamento cruzado posterior
equimosis y aumento de volumen en relación a la cara C) LIgamento colateral medial D) Menisco medial E) Tendón
lateral del tobillo, que aumenta especialmente a la de la pata de ganso
compresión de los maleolos. El diagnóstico más probable A
es: A) Esguince de tobillo B) Fractura de tobillo C) Luxación 129
de tobillo D) Rotura de la sindesmosis tibio-peroneal E) El tratamiento antibiótico de la artritis séptica suele ser: A)
Lesión del cartílago articular Cloxacilina endovenosa B) Gentamicina endovenosa C)
B Vancomicina endovenosa D) Ceftriaxona endovenosa E)
121 Cefazolina + metronidazol endovenosos
Un paciente de 45 años luego de levantar unas cajas, A
presenta dolor intenso en la zona cervical posterior, que se 130
irradia por el hombro y la extremidad superior derecha. Al Un paciente de 40 años consulta por dolor en el codo
examen físico no se aprecia déficit sensitivo ni motor, sin derecho, que aparece con algunos movimientos. Al
embargo el dolor impide los movimientos de flexo extensión examinarlo se constata dolor a la palpación de la inserción
del cuello. La conducta más adecuada es: A) Solicitar TAC de los músculos extensores de la muñeca. E diagnóstico
de cuello B) Solicitar RMN de cuello C) Solicitar Rx de cuello más probable es: A) Bursitis B) Codo del golfista C) Codo del
D) Indicar reposo absoluto, analgésico y solicitar ecografía tenista D) Tendinitis bicipital E) Tendinitis tricipital
de cuello E) Indicar reposo relativo y analgésicos, sin C
necesidad de imágenes 131
E Una paciente de 20 años sufre una caída a nivel,
122 apoyándose sobre la mano derecha, evolucionando con
dolor a nivel del codo y la zona proximal del antebrazo. Al 140
examen físico se observa dolor a los movimientos de Una niña de 3 meses de edad es diagnosticada de displasia
pronosupinación de la extremidad superior derecha. El de cadera. La conducta más adecuada es: A) Tratar con
diagnóstico más probable es: A) Pronación dolorosa B) ejercicios posturales y de fortalecimiento de la musculatura
Luxofractura de codo C) Fractura de cúpula radial D) posterior B) Tratar con correos de Pavlik C) Tratar con un
Fractura de antebrazo E) Fractura supracondílea de humero yeso pelvipedio D) Tratar de manera quirúrgica E) Enviar a
C domicilio y explicar que se corregirá sola, sin necesidad de
132 tomar ninguna medida
Un paciente sufre una fractura del cuello femoral. ¿Qué B - Tratar con correos de Pavlik
complicación aparecerá con mayor probabilidad? A) 141
Necrosis avascular B) Artrosis de cadera C) Lesión de nervio Una mujer de 17 años presenta traumatismo en la rodilla
ciático D) Lesión del nervio femoral E) Lesión de la arteria izquierda, siendo diagnosticada de esguince de rodilla, el
femoral que se maneja con reposo y antiinflamatorios. Evoluciona
A con dolor persistente, por lo que consulta nuevamente. Se
133 solicita una radiografía de rodilla que muestra reacción
Un niño de 8 años sufre una fractura de antebrazo en tallo perióstica y compromiso de partes blandas. El diagnóstico
verde, la que se maneja ortopédicamente. Días después más probable es: a) Osteomielitis del fémur distal b)
presenta intenso dolor, que no responde a la analgesia Osteosarcoma osteogénico c) Fractura del fémur distal, con
habitual. Al examen físico presenta pérdida de los vellos de callo óseo d) Osteocondroma e) Hematoma en organización
la mano e intenso dolor de tipo quemante, que aumenta con B - Osteosarcoma osteogénico
los movimientos de la muñeca y los dedos. El diagnóstico 142
más probable es: A) Distrofia simpático refleja B) Síndrome Una paciente de 30 años, con antecedente de displasia de
compartimental C) Trombosis venosa profunda D) cadera derecha, tratada con cirugía abierta en su infancia,
Atrapamiento del nervio mediano E) Retraccion isquémica consulta por dolor muy intenso en la cadera izquierda, que
de Volkman la hace cojear permanentemente. Al examen físico presenta
A acortamiento de 25 mm de la extremidad inferior derecha y
134 su radiografía muestra artrosis severa de la articulación.
Un paciente de 29 años presenta dolor en la rodilla ¿Cuál es la conducta más adecuada? a) Tratamiento
izquierda, luego de una excursión a una montaña. Al sintomático de la artrosis b) Indicar tratamiento
examen se aprecia movilidad completa, pero dolor a la kenesioterapéutico c) Realizar osteotomía acetabular d)
palpación de la zona proximal medial de la tibia izquierda. El Instalar prótesis total de cadera e) Compensar el
diagnóstico más probable es: A) Lesión del menisco medial acortamiento con calzado con realce
B) Esguince del ligamento colateral medial C) Enfermedad D - Instalar prótesis total de cadera En mi opinión tiene
de Osgood Schlatter D) Tendinitis de la pata de ganso E) indicación de reemplazo articular. Pero la decisión exacta
Lesión del tendón de la fascia lata corresponde al especialista, según cada caso.
D 143
135 Un niño es diagnosticado de displasia de cadera a los 6
Un paciente sufre una fractura de muñeca, con rasgo meses de vida, mediante una radiografía, que muestra la
intraarticular. La cmplicación asociada a esta lesión es: A) displasia, con luxación del extremo proximal del fémur
Artrosis B) Lesión del nervio mediano C) Lesión de la arteria izquierdo. ¿Cuál es el tratamiento de elección? a) Doble
radial D) Necrosis avascular E) Síndrome compartimental pañal b) Correas de Pavlik c) Yeso con fijador d) Osteotomía
A periacetabular tipo Ganz
136 B - Correas de Pavlik Tiene una displasia de cadera menor a
Un paciente de 30 años presenta dolor en la zona media del 9 meses, que se trata con arnés de Pavlik.
muslo. Se solicita una radiografía que muestra un tumor de 144
3cm de diámetro, ubicado en la zona periférica de la diáfisis Una paciente sufre una fractura de la diáfisis humeral
femoral, con un centro radiolúcido y una periferia más derecha, la que es manejada mediante cirugía, con
radiopaca que el resto del hueso. El diagnóstico más osteosíntesis con placa. A las 24 horas, evoluciona con dolor
probable es: A) Tumor de células gigantes B) Osteoma en el sitio de la fractura, asociada a imposibilidad de
osteoide C) Osteosarcoma D) Osteocondroma E) Quiste extender la muñeca derecha. Mantiene la flexión activa de
óseo simple los dedos de esa mano. ¿Cuál es el diagnóstico más
B probable? a) Trombosis de la arteria braquial b) Neurapraxia
137 del nervio radial c) Desplazamiento de la placa d) Síndrome
Un niño de 1 mes de edad presenta signo de Barlow y se compartimental e) Lesión del plexo braquial
constata limitación a a abducción de la cadera izquierda. La B - Neurapraxia del nervio radial Confundía el hecho de
conducta más adecuada es: A) Solicitar radiografía AP de haber dolor en la zona, lo que orientaba a un síndrome
pelvis B) Solicitar radiografía de la cadera izquierda C) compartimental, pero solo se describía la afectación del
Solicitar ecografía de cadera D) Solicitar RMN de caderas E) nervio radial, sin otras alteraciones, por lo que lo más
Resolver quirúrgicamente de inmediato probable es que haya sido la neurapraxia.
C 145
138 Un paciente de 24 años, sufre una caída a nivel, mientras
Un paciente sufre una "torsión del pie izquierdo" mientras jugaba básquetbol, cayendo con el hombro izquierdo, contra
caminaba, con inversión forzada, evolucionando con dolor al el suelo. Presenta impotencia funcional y al examen físico
caminar. Al examen físico e aprecia equimosis y aumento de destaca dolor a la movilización del hombro izquierdo. Se
volumen en relación a la cara lateral del tobillo, con dolor a solicita una radiografía AP de hombro y axial de escápula,
la inversión del pie. El diagnóstico más probable es: A) que se muestran a continuación: FOTO: Se ve ligera
Esguince de tobillo B) Fractura de tobillo C) Luxación de separación entre la clavícula distal derecha y el acromion,
tobillo D) Rotura de la sindesmosis tibio-peroneal E) Lesión con desplazamiento hacia superior del extremo distal de la
del cartílago articular clavícula. El diagnóstico más probable es: a) Luxación
A anterior de hombro b) Fractura de escápula c) Luxación
139 acromioclavicular d) Fractura de clavícula e) Lesión del
Un paciente de 75 años presenta intenso dolor en la zona tendón del manguito rotador
lumbar desde hace 2 meses, que se irradia por la C - Luxación acromioclavicular En la imagen se veía la
extremidad inferior derecha. Al examen físico se aprecia disyunción acromioclavicular, con leve desplazamiento
debilidad asimétrica de las extremidades inferiores. La hacia superior del extremo distal de la clavícula.
conducta más adecuada es: A) Iniciar analgésicos y solicitar 146
ecografía lumbar B) Iniciar analgésicos y solicitar RMN de Un paciente de 30 años sufre una caída de altura, desde 15
columna C) Iniciar analgésicos y solicitar Rx lumbar D) metros, golpéandose contra el suelo. Al examen físico está
Indicar reposo absoluto, analgésicos, sin necesidad de orientado, con FC: 80x’ y PA: 120/80 mmHg, con mucho
imágenes E) Indicar reposo relativo y analgésicos, sin dolor a la compresión de la pelvis, mayor a izquierda. Se
necesidad de imágenes solicita una radiografía de pelvis AP, que se muestra a
B continuación: FOTO: se ve una fractura de pelvis, con
compromiso del acetábulo izquierdo y algún grado de IMAGEN. ¿Cuál es el diagnóstico más probable? a)
separación en la sínfisis del pubis. ¿Cuál es la conducta Osteocondroma b) Condrosarcoma c) Tumor de células
inicial más adecuada? a) Pasar una sonda Foley b) Indicar gigantes d) Sarcoma de Ewing e) Sarcoma de partes
reposo en hamaca c) Administrar antibióticos d) Realizar blandas FOTO: Radiografía con lesión osteolítica en el tercio
cirugía e) Solicitar ecografía de abdomen y pelvis medio de la diáfisis el fémur, con un tumor alrededor,
D - Realizar cirugía ¿D? Pregunta muy mala. Lo primero apenas visible. RMN muestra un gran tumor, en relación a la
frente a una fractura de pelvis, y frente a un diáfisis femoral, de gran tamaño, heterogéneo.
politraumatizado en general, es hacer el ABC y en este D - Sarcoma de Ewing Por la ubicación (tumor diafisiario), el
caso, instalar vías venosas periféricas y aportar volumen. aspecto (maligno: con disrupción de la cortical y
Luego, lo principal es reducir de manera externa la pelvis, heterogéneo) y la edad es probable que sea un sarcoma de
de modo de evitar o al menos limitar las hemorragias Ewing. Sin embargo un sarcoma de partes blandas sí podría
pélvicas, lo que se puede hacer con un tutor externo y si no verse así, pero el cuadro es clásico de un Sarcoma de
se tiene, son una férula neumática o por último con una Ewing. Además en todas las demás alternativas hay
sábana, pero la opción de hamaca no orienta a eso (además tumores óseos, que tienen otros aspectos y aparecen en
que no hay hamacas en los servicios de salud). La única edades distintas.
opción razonable era la cirugía de reducción, pero al 152
preguntar la conducta inicial, parecería que la respuesta es Un paciente de 45 años presenta dolor lumbar muy intenso,
otra. Respecto a la sonda Foley, es posible ponerla, pero la de inicio súbito, que se irradia por la extremidad inferior
indicación es la retención urinaria y en la fractura de pelvis, izquierda. Al examen físico presenta debilidad a la flexión
hay riesgo de rotura uretral, lo que contraindica la Sonda plantar del primer ortejo izquierdo, disminución de la
Foley. sensibilidad de la zona lateral del pie izquierdo y abolición
147 del reflejo aquiliano ipsilateral. ¿Cuál es el diagnóstico más
Una paciente de 65 años sufre caída a nivel, apoyando la probable? a) Síndrome facetario, lateralizado a izquierda b)
mano derecha contra el suelo, en extensión de la Lesión del nervio ciático izquierdo c) Afectación de la raíz S1
extremidad. Evoluciona con dolor intenso en la muñeca, que d) Lesión del plexo lumbar e) Afectación de la raíz L5
le impide realizar movimientos. Al examen físico se aprecia C - Afectación de la raíz S1 Es el cuadro clásico de la lesión
deformidad de la muñeca y dolor epicrítico a la presión del de S1.
radio distal. El diagnóstico más probable es: a) Fractura de 153
escafoides b) Fractura de epifisis distal del radio c) Un paciente presenta dolor en la cara lateral de la cadera
Disyunción-fractura de radio distal d) Luxofractura de derecha, que aumenta con la marcha y al recostarse
Monteggia e) Luxación de cúpula radial apoyado en esa cadera. Los movimientos están
B - Fractura de epifisis distal del radio ¿B, C?. No hay duda conservados. ¿Cuál es el diagnóstico más probable? a)
de que es una fractura de radio distal, pero la clínica es Síndorme de la banda ileotibial b) Bursitis ileopectínea c)
insuficiente para distinguir estas dos opciones. Yo marqué la Bursitis trocantérica d) Pinzamiento femoroacetabular e)
B. Artrosis de cadera
148 C - Bursitis trocantérica Cuadro clásico
Un paciente de 18 años, consulta porque desde hace un año 154
presenta aumento de volumen progresivo en la cara Un paciente de 77 años presenta dolor inguinal derecho de
anterior del muslo derecho, que recientemente se ha vuelto 2 años de evolución, que aumenta con la marcha,
dolorosa. Al examen físico se palpa una masa de 10 cm de limitándola. Al examen físico se aprecia pérdida de la
diámetro, de consistencia dura, no pétrea, sin afectación de rotación interna y la flexión de la cadera derecha. El
la piel. ¿Cuál es la conducta más adecuada? a) Realizar diagnóstico más probable es: a) Necrosis avascular de la
resección y biopsia b) Observar evolución c) Solicitar cabeza femoral b) Coxartrosis c) Hernia foraminal S1 d)
ecografía de partes blandas d) Solicitar radiografía de fémur Pinzamiento femoroacetabular e) Estenosis raquimedular
e) Realizar punción y drenaje B - Coxartrosis Por llevar dos años orienta a artrosis
C - Solicitar ecografía de partes blandas Es un probable (coxartrosis = artrosis de cadera). Alguien anotó 2 semanas
sarcoma de partes blandas. Debe estudiarse con resonancia en FB: sería la necrosis avascular.
magnética nuclear, para determinar el grado de invasión, 155
antes de definir qué tipo de cirugía se le hará. Como no Un paciente, luego de un “sueño profundo”, en relación a
aparecía en las opciones, lo más razonable es hacer la una intoxicación alcohólica, presenta debilidad y dificultad
ecografía, por la misma razón. Luego sí se hará la resección para tomar objetos con la mano derecha, como sostener
y biopsia. Es una mala pregunta. una taza, la que se le cae. Al examen físico no tiene
149 alteraciones en la sensibilidad, ni en los reflejos
Niño de 12 años, obeso, refiere dolor de 3 meses en cadera osteotendíneos, los que permanecen conservados. Sin
izquierda y claudicación progresiva, que actualmente embargo, al solicitarle que ponga los brazos en posición
impide la marcha. Al examen físico presenta dolor a la horizontal, se observa que la mano derecha está “caída”.
rotación interna y externa de la cadera izquierda. Se realiza ¿Cuál de los siguientes es el diagnóstico más probable? a)
adiografía AP pelvis y caderas, que se muestra a Lesión del nervio radial b) Lesión del plexo braquial c)
continuación: FOTO: se muestra una epifisiolisis de cadera Lesión de los nervios cubital y mediano d) Isquemia de los
izquierda clásica. El diagnóstico es: a) Displasia del músculos del antebrazo e) Accidente vascular encefálico
desarrollo de la cadera b) Epifisiolisis c) Fractura del cuello A - Lesión del nervio radial Es la historia clásica de la
del fémur d) Luxacion de la Cadera izquierda e) Necrosis neuropatía radial por compresión, del borracho. Igual la
avascular de la cabeza del fémur dificultad para tomar objetos podría orientar a lesión del
B - Epifisiolisis Es una clásica epifisiolisis de la cabeza plexo braquial.
femoral. 156
150 Un niño de 3 meses de edad se realiza una radiografía de
Usted examina a un niño de 4 años de edad, que presenta pelvis, que muestra displasia de cadera izquierda, con
pie plano valgo flexible bilateral. El resto de su examen índice acetabular de 36 grados. La indicación más adecuada
físico es normal. ¿Cuál es la conducta más adecuada? a) es: a) Ejercicios de rotación y compresión b) Bota larga
Controlar en un año b) Realizar radiografía de ambios pies bilateral de yeso, con yugo c) Yeso pelvipedio d) Uso de
c) Indicar plantillas correctoras d) Realizar resonancia doble pañal e) Arnés de Pavlik
magnética nuclear de ambos pies e) Indicar calzado E - Arnés de Pavlik Menores de 10 meses (según Minsal) o
corrector menores de 6 meses (universalmente aceptado) se manejan
A - Controlar en un año El pie plano flexible se observa. Solo con correas de Pavlik
se indican plantillas si hay dolor al caminar (no son 157
correctoras, sino tratamiento sintomático). Un adolescente de 16 años presenta dolor en la rodilla
151 derecha, luego de haberse golpeado. Al examen físico se
Un adolescente de 14 años presenta dolor en la rodilla palpa aumento de volumen de consistencia ósea, por lo que
derecha. Al examen físico se aprecia aumento de volumen y se solicita una radiografía, que muestra una lesión ósea
dolor, en relación al fémur derecho. Tiene antecedente de herterogénea, ubicada en el extremo distal del fémur, con
traumatismo en la zona. Se solicitan exámenes radiológicos disrupción de la cortical y zonas líticas, con otras de
(radiografía y resonancia magnética), que se muestran en la aumento de la densidad. El diagnóstico más probable es: a)
Osteocondroma b) Tumor de células gigantes c) por VHA E) Infección por estreptococo hemolítico del grupo
Osteosarcoma d) Displasia ósea fibrosa e) Quiste óseo A
C - Osteosarcoma Tumor óseo de aspecto maligno. Por C - Infección por VEB PTI: sindrome purpúrico
ubicación y edad es osteosarcoma (adolescentes y adultos trombocitopénico en ausencia de patologías de base e
mayores). Recordar que Sarcoma de Ewing es diafisiario. infecciosas, con trombocitopenia ASLO: usualmente
158 confirma infeccion por EBH cuando es por encima de 200
Un paciente de 27 años sufre una fractura cerrada de CMV es muy similar, causa un sindrome mononucleosico.
antebrazo, que es manejada con un valva de yeso Algunas diferencias: - En el síndrome de mononucleosis por
braquiopalmar. A las 5 horas presenta aumento del dolor y CMV predominan los síntomas sistémicos y la fiebre. Los
edema de la extremidad. El diagnóstico más probable es: a) signos de aumento de tamaño ganglionar y la
Trombosis venosa profunda b) Síndrome compartimental c) esplenomegalia son menos comunes que los vistos con VEB.
Distrofia simpático refleja d) Alergia al yeso e) Síndrome de - A diferencia del VEB, el CMV pocas veces causa
túnel carpiano agudo faringoamigdalitis exudativa. - Los adultos con
B - Síndrome compartimental Por el corto tiempo de mononucleosis por CMV suelen tener mayor edad que
evolución y el dolor es síndrome compartimental. Además aquellos con infección por VEB.
es clásico de las Fx de antebrazo. Falta información
159
Un hombre protagoniza un accidente de tránsito, chocando
de manera frontal, contra otro automóvil. Presenta intenso
dolor en la cadera derecha, que le impide caminar. En su
examen tiene una rotación fija hacia interno, con aducción
del muslo. El diagnóstico más probable es: a) Fractura de
cuello femoral b) Fractura de cotilo c) Esguince grado 3 de
cadera d) Luxación anterior de cadera e) Luxación posterior
de cadera
E - Luxación posterior de cadera Es una luxación posterior
clásica, con la posición impúdica.
160
Un paciente de 28 años sufre una caída a nivel, cayendo
sobre la mano izquierda, con extremidad superior izquierda
en extensión. Al examen físico presenta dolor a la
compresión axial del pulgar y al presionar la tabaquera
anatómica. ¿Cuál es el diagnóstico más probable? a)
Fractura de la base de la primera falange proximal b)
Fractura de escafoides c) Fractura perilunar d) Fractura del
extremo distal del radio e) Esguince de la primera
articulación metacarpofalángica
B - Fractura de escafoides Es un fractura de escafoides
clásica.
161
Un paciente sufre una caída a nivel, con golpe en el hombro
derecho, evolucionando con dolor e impotencia funcional.
Se realizan radiografías de hombro, una de las que se
muestra a continuación: FOTO (Radiografía AP de hombro,
con fractura del tercio medio de la clavícula derecha, con
ligero desplazamiento, sin conminución) ¿Qué tipo de
inmovilización es la más adecuada? a) Vendaje axial de
hombro b) Vendaje en ocho c) Yeso Velpeau d) Yeso
toracobraquial e) Cabestrillo
E - Cabestrillo E, ¿B?. El cabestrillo y el vendaje en 8 son los
más usados en las fracturas del tercio medio de clavícula.
Sin embargo, también se pueden usar otros tipos de
inmovilización, según la valoración por el especialista. El
vendaje en 8 se asocia a mayores molestias, por
compresión cutánea y del complejo neurovascular axilar, así
que el cabestrillo suele ser lo más recomendado. De todos,
modos, es una pregunta de especialista.
Una paciente de 31 años, secretaria, consulta por dolor en
el hombro derecho, que ha aumentado en el último tiempo.
Ella refiere que es más intenso en la noche y que aparece
especialmente al levantar el codo por sobre el hombro.
¿Cuál es el diagnóstico más probable? a) Tendinitis del
manguito rotador b) Hernia del núcleo pulposo cervical c)
Artrosis glenohumeral d) Tendinitis de la cabeza larga del
bíceps e) Capsulitis adhesiva de hombro
A - Tendinitis del manguito rotador Era una tendinitis del
manguito rotador clásica.
Mujer de 17 años sin hábitos tóxicos, que cinco días antes
de su ingreso comienza con fiebre, odinofagia, cansancio
extremo y dolorimiento sordo en hipocondrio izquierdo. Su
médico ha indicado tratamiento con amoxicilina dos días
antes del ingreso. EF: febril y deshidratada, rash macular
con componente petequial discreto en brazos, abdomen y
piernas; linfadenopatía cérvico-axilar, fauces edematosas
con exudados blanquecinos, hepatoesplenomegalia
moderada, petequias en paladar. Lab: leucocitos 70% linfo,
18% mono, 3% neutro, plq 70000/mm3, GOT y GPT tres
veces su valor máximo normal. Frotis: células
mononucleares atípicas. ASLO menor de 100 U. Diagnóstico
de esta paciente: A) Infección por CMV B) Púrpura
trombopénica idiopática C) Infección por VEB D) Infección

S-ar putea să vă placă și